You are on page 1of 68

EgyptianPediatrics

Yahoo Group
http://health.groups.yahoo.com/group/
EgyptianPediatrics/

Commentary
Zika Virus Pediatricians Be Aware

AUTHOR DISCLOSURE Dr Rathore has


disclosed no nancial relationships relevant to
this article. This commentary does not contain
a discussion of an unapproved/investigative
use of a commercial product/device.

Every few years, a new (not really) funnysounding infectious disease is in the news
and causing anxiety: rst it was SARS
(severe acute respiratory syndrome), then
avian u, swine u, dengue, MERS (Middle
East respiratory syndrome), chikungunya,
Ebola, and now in 2016 its Zika virus.
Zika virus was rst identied in 1947 at
the East African Virus Research Institute (now the Uganda Virus Research
Institute) in Entebbe, Uganda, as a cause of febrile illness in rhesus macaques.
(1) Until 2007, Zika virus caused only rare cases of human disease in Africa and
Southeast Asia. However, in April 2007, an outbreak was reported on Yap Island
that subsequently spread to other Polynesian islands. This was followed in 2015
by an explosive and widespread outbreak in South and Central America that
is ongoing. Brazil seems to be particularly severely affected.
Zika virus is transmitted by Aedes mosquitoes, the same mosquito that transmits
Dengue and Chikungunya viruses. The Aedes mosquitoes that are known to transmit
or can potentially transmit Zika virus are present in 32 States (2). Although so far
no autochthonous cases of Zika virus transmitted by mosquitoes have been
diagnosed in the United States, one sexually transmitted case of Zika virus has
been identied in the United States during the current pandemic. (3)
The incubation period for Zika virus infection is 2 to 14 days. The disease
has a wide spectrum and only 1 in 5 infected patients becomes symptomatic.
Hospitalizations are uncommon and death is rare. Clinically, Zika virus infection presents similarly to many other viral infections, with fever (often lowgrade), vomiting, maculopapular rash, arthralgias, myalgias, retro-orbital pain,
and conjunctivitis.
Serologic diagnosis is not dependable because of potential cross-reactivity
with dengue and chikungunya viruses. Polymerase chain reaction that can detect
the RNA of Zika virus is available from the Centers for Disease Control and
Prevention (CDC) and some state health departments.
There is no commercially available test for Zika virus and no specic antiviral
treatment; management is primarily supportive. There is also no vaccine to
protect against Zika virus infection. Prevention is largely dependent on avoidance
of areas where there is active Zika virus transmission (Figure) as well as mosquito
control and measures to prevent mosquito bites.
Compared to previous new emerging infections, Zika virus infection
has particular interest for pediatricians because of the major concern that
such infection may be responsible for microcephaly in infants born to infected
women. Although no causal relationship has been determined between Zika
virus infection during pregnancy and microcephaly in the newborn, the manyfold increase in cases of microcephaly in the midst of a Zika virus epidemic

Vol. 37 No. 4
Downloaded from http://pedsinreview.aappublications.org/ by guest on April 2, 2016

APRIL 2016

133

Figure. Areas of reported Zika virus transmission.

offers compelling epidemiologic suggestion of a link. (4) A


total of 2,401 suspected cases of microcephaly have been
reported in Brazil during the period of outbreak. Of these,
134 were conrmed as being related to Zika virus infection,
102 were considered not related, and 2,165 are still under
investigation. (5) Further careful research is needed to
determine if this temporal association is causative.
Because of this concern, the CDC recommends that
pregnant women avoid travel to areas of ongoing Zika virus
transmission. If travel is necessary, measures should be
taken to prevent mosquito bites. Pregnant women returning
from areas of Zika virus activity should consider testing to
determine if they have become infected. (6)
This new viral infection is another reminder that world
is becoming smaller, and infections once exotic and far off
can reach our shores quickly and sometimes stealthily. We
need to be vigilant in identifying potential emerging infection threats quickly. Building public health infrastructure in
under-resourced parts of the world benets not just local
populations but those of us in the resource-rich parts of the
world.

Mobeen H. Rathore, MD, CPE


Editorial Board member
University of Florida Center for HIV/AIDS Research,

Education and Service (UF CARES)


Jacksonville, FL

References
1. Dick GWA, Kitchen SF, Haddow AJ. Zika virus. I. Isolations
and serological specicity. Trans R Soc Trop Med Hyg. 1952;
46(5):509520
2. Fauci AS, Morens DM. Zika virus in the Americas yet another
arbovirus threat. N England J Med. 2016;374(7):601604
3. Oster AM, Brooks JT, Stryker JE, et al. Interim guidelines for
prevention of sexual transmission of Zika virus - United States, 2016.
MMWR. 2016;65(5):120121, DOI: http://dx.doi.org/10.15585/
mmwr.mm6505e1
4. Oliveira Melo AS, Malinger G, Ximenes R, Szejnfeld PO, Alves
Sampaio S, Bispo de Filippis AM. Zika virus intrauterine infection
causes fetal brain abnormality and microcephaly: tip of the iceberg?
Ultrasound Obstet Gynecol. 2016;47(1):67
5. European Centre for Disease Prevention and Control. Epidemiological
Update: Outbreaks of Zika Virus and Complications Potentially Linked to
the Zika virus infection. 2015. Available at: http://ecdc.europa.eu/
en/press/news/_layouts/forms/News_DispForm.aspx?ID
1342&List8db7286c-fe2d-476c-9133-18ff4cb1b568&Sourcehttp%3A
%2F%2Fecdc%2Eeuropa%2Eeu%2Fen%2Fpress%2Fepidemiological
%5Fupdates%2FPages%2Fepidemiological%5Fupdates%
2Easpx#sthash.oX5TQfDj.dpuf. Accessed February 5, 2016
6. Petersen EE, Staples JE, Meaney-Delman D, et al. Interim
guidelines for pregnant women during a Zika virus outbreak
United States, 2016. MMWR Morb Mortal Wkly Rep. 2016;65
(2):3033

Parent Resources from the AAP at HealthyChildren.org


https://www.healthychildren.org/English/ages-stages/prenatal/Pages/Zika-Virus.aspx
Spanish: https://www.healthychildren.org/Spanish/ages-stages/paginas/Pages/Zika-Virus.aspx

134

Pediatrics in Review
Downloaded from http://pedsinreview.aappublications.org/ by guest on April 2, 2016

Zika Virus Pediatricians Be Aware


Mobeen H. Rathore
Pediatrics in Review 2016;37;133
DOI: 10.1542/pir.2016-0003

Updated Information &


Services

including high resolution figures, can be found at:


http://pedsinreview.aappublications.org/content/37/4/133

References

This article cites 5 articles, 1 of which you can access for free at:
http://pedsinreview.aappublications.org/content/37/4/133#BIBL

Subspecialty Collections

This article, along with others on similar topics, appears in the


following collection(s):
Infectious Disease
http://beta.pedsinreview.aappublications.org/cgi/collection/infectious
_diseases_sub
International Child Health
http://beta.pedsinreview.aappublications.org/cgi/collection/internatio
nal_child_health_sub
Public Health
http://beta.pedsinreview.aappublications.org/cgi/collection/public_he
alth_sub

Permissions & Licensing

Information about reproducing this article in parts (figures, tables) or


in its entirety can be found online at:
http://beta.pedsinreview.aappublications.org/site/misc/Permissions.x
html

Reprints

Information about ordering reprints can be found online:


http://beta.pedsinreview.aappublications.org/site/misc/reprints.xhtml

Downloaded from http://pedsinreview.aappublications.org/ by guest on April 2, 2016

Zika Virus Pediatricians Be Aware


Mobeen H. Rathore
Pediatrics in Review 2016;37;133
DOI: 10.1542/pir.2016-0003

The online version of this article, along with updated information and services, is
located on the World Wide Web at:
http://pedsinreview.aappublications.org/content/37/4/133

Pediatrics in Review is the official journal of the American Academy of Pediatrics. A monthly
publication, it has been published continuously since 1979. Pediatrics in Review is owned,
published, and trademarked by the American Academy of Pediatrics, 141 Northwest Point
Boulevard, Elk Grove Village, Illinois, 60007. Copyright 2016 by the American Academy of
Pediatrics. All rights reserved. Print ISSN: 0191-9601.

Downloaded from http://pedsinreview.aappublications.org/ by guest on April 2, 2016

Hematopoietic Stem Cell Transplantation in


Children and Adolescents
Gregory M.T. Guilcher, MD*
*Section of Pediatric Oncology/BMT, Alberta Childrens Hospital; Departments of Oncology and Pediatrics, University of Calgary, Calgary, Alberta, Canada.

Educational Gap
Hematopoietic stem cell transplantation (HSCT) indications and practices
have changed signicantly over the last 20 years. Evolving hematopoietic
stem cell sources, less toxic conditioning regimens, and improving graftversus-host disease prophylaxis and therapy have broadened the
application of HSCT from malignant conditions to increasing numbers of
nonmalignant diseases.

Objectives

After completing this article, the reader should be able to:

1. Understand general principles of allogeneic and autologous


hematopoietic stem cell transplantation (HSCT), including the variety of
hematopoietic stem cell sources.
2. Discuss the variability in intensity of current conditioning approaches,
which inuences the risks and applicability of HSCT.
3. Recognize that HSCT involves acute and chronic complications and the
importance of general clinicians and subspecialists in their
management.
4. Review the pathophysiology of graft-versus-host disease, its
presentation, and its prevention and management.
5. Identify the increasing number of nonmalignant indications for HSCT in
children such that children who might benet from this procedure are
considered for timely referral as appropriate.

CASE STUDY

AUTHOR DISCLOSURE Dr Guilcher has


disclosed no nancial relationships relevant to
this article. This commentary does contain a
discussion of an unapproved/investigative
use of a commercial product/device.

A 1-year-old child is referred to your ofce for a developmental assessment due to


delayed speech and gross motor skills. You notice coarse facial features and on
physical examination document corneal clouding, hepatosplenomegaly, and
numerous skeletal deformities. You suspect a metabolic disorder and request
an urgent referral to a metabolic specialist. The specialist clinically diagnoses
Hurler syndrome (mucopolysaccharidosis IH) and conrms a-L-iduronidase
deciency with urinary glycosaminoglycan testing and subsequently by enzyme

Vol. 37 No. 4
Downloaded from http://pedsinreview.aappublications.org/ by guest on April 2, 2016

APRIL 2016

135

deciency in broblasts. While genetic testing results are


pending, you discuss the case with the metabolic specialist
and agree that an urgent referral to a pediatric hematopoietic
stem cell transplantation (HSCT) specialist is warranted
before genetic testing results are available. The best neurologic outcomes are seen when HSCT is performed as soon
as possible, preferably before age 2 years. Having general
knowledge about HSCT planning and complications, you
help the family prepare for their meeting with the pediatric
HSCT specialist, allowing for a more productive consultation, and offer to share ongoing care of the child both before
HSCT and during subsequent follow-up.

NOMENCLATURE
HSCT is the procedure of infusing blood stem cells from a
donor into a recipient. When the donor and recipient are
different people, the procedure is termed an allogeneic
HSCT; if the donor and recipient is the same person, it is
an autologous HSCT. Syngeneic HSCT describes a donation
between identical twins.
Hematopoietic stem cells (HSCs) may be collected from
bone marrow, peripheral blood, or the umbilical cord/
placental unit of a newborn (UCB).
Human leukocyte antigens (HLAs) are tested at major
histocompatibility loci: Class I (A, B, and C) and Class II
(DR; DQ in some centers). At least 6 loci routinely are
analyzed for a UCB product and 8 to 10 loci for a live donor
product (ie, bone marrow or peripheral blood). The degree
of matching is expressed as the numerator of matched loci
over the denominator of loci tested. HLA matching may be
tested at low (antigenic), medium, or high (allelic) levels of
resolution.
Graft-versus-host disease (GVHD) is a serious and potentially life-threatening complication of HSCT in which the
donor T cells cause an inammatory response in the recipient tissues. This complication is described in detail later,
but the risk of its development has been historically reduced
by the best possible HLA matching at major loci as well as
the use of a related donor due to closer matching at untested
minor histocompatibility antigens. Newer approaches to
haploidentical HSCT (see denition later) and novel GVHD
prevention strategies, however, are reducing GVHD rates,
even in the setting of greater HLA disparity.
Allogeneic HSC donors are further characterized in
terms of the relationship between the donor and recipient
(Table 1). Fully matched related donations can come from a
minor or adult sibling or rarely a parent (often with a history
of consanguinity). Haploidentical HSCT involves donation
from a rst-degree relative (usually a mother) who shares 1

136

TABLE 1.

Nomenclature for Hematopoietic


Stem Cell Donors

Matched Sibling Donor

MSD

Mismatched Sibling Donor

MMSD

Matched Familial Donor (eg, parent)

MFD

Mismatched Familial Donor

MMFD

Matched Unrelated Donor

MUD

Mismatched Unrelated Donor

MMUD

Matched indicates all tested human leukocyte antigen (HLA) loci are the
same between donor and recipient.
Mismatched means at least 1 HLA locus differs between donor and
recipient (at either allelic or antigenic level of testing).

haplotype, typically matched at 5 to 8 of 10 HLA loci.


Unrelated HSC products may come from UCB donations
or a living adult donor (not minors).
Conditioning refers to the preparative chemotherapy,
immunotherapy, and/or radiotherapy given to a recipient
before stem cell infusion to facilitate engraftment of allogeneic donor HSCs and to prevent rejection. In this setting,
the HSCs are a primary component of the curative therapy;
in autologous HSCT, the conditioning is the actual therapy
and the HSCs are administered to rescue the hematopoietic
system. Myeloablative conditioning refers to intensive chemotherapy and/or radiation doses sufcient to cause bone
marrow aplasia in the absence of HSC infusion. Reducedintensity conditioning (RIC) describes nonmyeloablative or
less intensive conditioning regimens.
HSCs for UCB and autologous donation must be cryopreserved, whereas most allogeneic live donor products are
donated during the conditioning of the recipient. Allogeneic
products may also be manipulated to reduce plasma, red
blood cells, or T cells, depending on the donor-recipient
blood group matching/mismatching, the stem cell source,
the routine practices of the HSCT center, and the indication
for HSCT.

TRENDS IN PRACTICE
Internationally, more than 2,000 allogeneic HSCTs were
reported to have been performed in recipients younger than
age 20 years in 2012. (1) The use of UCB has increased over
the last 20 years, as have donations from unrelated live
donors. These trends are affected by improvements in
supportive care (including GVHD prevention and treatment) as well as donor availability, with expanded live donor
and UCB registries.

Pediatrics in Review
Downloaded from http://pedsinreview.aappublications.org/ by guest on April 2, 2016

RIC was developed for older recipients who were ineligible for myeloablative conditioning due to comorbidities.
Its use has expanded to many nonmalignant indications for
children in whom a phenotype can be reversed with even
relatively low numbers of engrafted donor HSCs (mixed
donor chimerism) and there is a mix of hematopoietic cells
of donor and recipient origin. Several conditions, such as
severe combined immune deciency, hemophagocytic lymphohistiocytosis, and hemoglobinopathies, are known to be
cured with stable mixed-donor chimerisms as low as 20% to
30%. (2) The appeal of RIC lies in reduced rates of GVHD and
transplant-related mortality (TRM) in addition to fewer acute
and late toxicities due to lower doses of conditioning agents.
The increased use of RIC and haploidentical HSCT has
also inuenced the growing proportion of HSCT recipients
with nonmalignant diseases. This trend toward HSCT for
nonmalignant conditions is due to improved outcomes with
upfront non-HSCT childhood leukemia therapies as well as
advancements in the safety of HSCT. As the risks of morbidity and mortality decrease, the potential application of
HSCT as a curative option for various nonmalignant diseases broadens.
Expertise in haploidentical HSCT is increasing worldwide, particularly in Europe and the United States. Its appeal
lies in the almost universal availability of a donor, particularly for potential recipients whose HLA haplotypes are
underrepresented on existing volunteer registries. Risks
of GVHD and infection (due to T-cell depletion) as well
as required laboratory infrastructure complicate its application, but improved supportive care options have increased
the practice of haploidentical HSCT. Newer techniques such
as the use of cyclophosphamide after HSC infusion have
resulted in markedly improved rates of engraftment and
reduced rates of GVHD and infectious complications. (3)

PRINCIPLES OF HSCT
Allogeneic HSCT involves the replacement of the decient
recipient hematopoietic system with that of the donor. The
best possible HLA-matched donor is used, with a preference
for matched sibling, followed by matched related donors.
HLA testing and matching is currently limited to 8 to 10
major histocompatibility loci for living donors, yet minor
histocompatibility (H) antigens also inuence the risk of
GVHD. Minor H antigens are potentially immunogenic
peptides genetically coded outside of the major histocompatibility complex (MHC). (4) The coding loci for H antigens
are scattered throughout the genome in contrast to the
MHC being coded on chromosome 6. As a result, a related
fully HLA-matched donor is almost always preferred to an

unrelated donor with the same number of matched loci.


Unrelated donors may be identied through international
live donor registries or accredited public UCB banks. Identifying an unrelated donor and proceeding with HSCT
usually takes 1 or more months, depending on the rarity
of the recipient HLA-typing, donor availability to proceed
with donation, and medical clearance of both donor and
recipient. This process is generally shorter for UCB products because the donation has already been made and the
product has been cryopreserved.
Allogeneic stem cells can be donated as 1 of 3 stem cell
sources:
Bone marrow
Peripheral blood stem cells
Umbilical cord blood
Table 2 describes the method of donation as well as
advantages and disadvantages of each source of allogeneic
HSCs. Peripheral blood stem cells are less commonly used
in pediatric HSCT recipients due to higher risks of chronic
GVHD; they are typically only used for malignant indications or as part of a RIC protocol. Many considerations are
balanced in choosing a stem cell source: the recipients
underlying condition, the degree of HLA matching, the
urgency of the HSCT, the risk to the donor (particularly
for minor sibling donors who cannot consent for themselves), donor preference for method of donation, donor
health status (which may preclude a method of donation),
ABO status of donor and recipient, and size discrepancy
between donor and recipient. The stem cell dose (ie, number of donor HSCs) required for the HSCT recipient is
calculated based on recipient weight, which may not be
achievable based on the size of a prospective living donor.
Donations from living donors are almost always collected
within 2 days of infusion to ensure that the HSCT is not
subsequently cancelled due to a change in the recipients
eligibility status and to avoid cell loss with cryopreservation.
UCB products contain a xed number of cryopreserved stem
cells. A given UCB unit may have sufcient stem cells for a
smaller recipient but may be inadequate for a larger patient.
Additional considerations include the age of the donor,
the donor sex, and any pregnancies (if applicable). Younger
donors generally have more cellular bone marrows and
produce greater HSC yields. In addition, their donations
are associated with lower GVHD rates in recipients. Donations from females, particularly with increasing parity, are
associated with higher rates of GVHD. Male recipients with
female donors are at highest risk. (10)
Autologous stem cell collections are almost always from
peripheral blood, with bone marrow harvests usually
reserved for failed peripheral blood collections. Such

Vol. 37 No. 4
Downloaded from http://pedsinreview.aappublications.org/ by guest on April 2, 2016

APRIL 2016

137

TABLE 2.

Review of Hematopoietic Stem Cell Sources

HSC STEM CELL


SOURCE
Bone Marrow

METHOD OF COLLECTION

ADVANTAGES

DISADVANTAGES

Donor undergoes anesthesia, is


placed prone, and marrow
harvested bilaterally from iliac crests
Collection proceeds until donor
maximum volume collected
(10-20 mL/kg) or target HSC
dose achieved (whichever
comes rst)
Research underway regarding
benets of G-CSF administration
before donation to improve yield (5)

High engraftment rates

Pain after harvest for donor

Lower rates of chronic GVHD


compared to peripheral blood (6)

Donor size limits volume of marrow


that can be harvested (transfusion
of donor is discouraged)
High volumes of product can cause
volume overload for recipient
ABO incompatibility warrants
processing of sample to reduce
red blood cells and/or plasma
(HSC loss occurs with each
processing step)

Peripheral Blood

Donor receives G-CSF for 3-5 days


prior to donation
Apheresis catheter placement for
donor (often a femoral venous line
under anesthesia for pediatric donors)

High engraftment rates

1-2 days of donation on apheresis


machine (typically 4-8 hr/day)

May allow for lower-intensity


conditioning

Smaller donors unable to undergo


apheresis without blood product
exposure due to extracorporeal
blood volume (transfusion of
donor is discouraged)
Donor may not mobilize stem cells
peripherally (more common in
adult donors)

Collected after clamping of umbilical


cord blood
Method of collection should not
compromise mother or neonatal
donor in any way

Product can be procured


quickly for HSCT
HLA mismatching more permissive
(ie, 4-6/6 match can be used)
due to lower rates of GVHD

Higher rates of nonengraftment


(graft failure) (9)
Cell dose per recipient weight is
limited to existing cryopreserved
product (xed) and may be
lowered, depending on viability
before freezing and after thawing

Sample is processed and cryopreserved

May be superior for metabolic


disorders (8)
May obviate the need for minor
sibling donation if sibling UCB
available
No donor risk

Higher stem cell yields


Possibly lower relapse rates
for malignant diseases (6)(7)

Collection proceeds until target HSC


dose achieved (diminishing yield
with ongoing time on circuit)
Umbilical Cord
Blood

G-CSF exposure to donor can cause


bone pain
Ongoing concern over long-term
risks of G-CSF exposure to donor
bone marrow (although data
show no clear evidence of harm)
Higher rates of chronic GVHD (6)

Higher rates of viral infections


(delayed immune recovery) (9)
Cannot access additional HSCs if
nonengraftment or early relapse
for donor lymphocyte infusion
Medical history of donor generally
unknown

G-CSFgranulocyte colony-stimulating-factor, GVHDgraft-versus-host disease, HLAhuman leukocyte antigen, HSChematopoietic stem cell,
HSCThematopoietic stem cell transplantation, UCBumbilical cord blood

collections are typically timed at the point of initial hematopoietic recovery following myelosuppressive chemotherapy,
in combination with granulocyte colony-stimulating factor
(G-CSF). Steady-state collections can also be performed
with G-CSF administration alone. The HSCs are then

138

cryopreserved to be used later to rescue the patient following


high doses of chemotherapy or radiation, allowing for more
rapid hematopoietic recovery.
HSCs are infused into the recipient after conditioning
chemotherapy and/or radiation (see next section). Such cells

Pediatrics in Review
Downloaded from http://pedsinreview.aappublications.org/ by guest on April 2, 2016

are infused into the venous system using a central vascular


access device but may also be infused into a peripheral
intravenous catheter. No lters can be placed on the tubing,
which could block the HSCs from entering the circulation.
Premedication is required for cryopreserved products to avoid
reaction to the preservative required for the cells to tolerate
freezing, and such premedication is also used for ABO
incompatibilities with bone marrow products. The HSCs
enter the marrow via adhesion molecule recognition and
start to grow and mature immediately. However, 2 to 3 weeks
generally is required for measurable neutrophil counts (or
engraftment) and for red blood cell and platelet transfusion
independence. The fastest rates of HSC engraftment are seen
with autologous rescues and with peripheral blood stem cell
products; UCB products are often the slowest to engraft. (9)

CONDITIONING FOR HSCT


Conditioning, or the preparative regimen, refers to the
combination of chemotherapy, immunotherapy, and/or
radiation therapy given to an HSCT recipient before the
HSC infusion. Such conditioning is usually administered
over 1 to 2 weeks before HSC infusion. Immune suppression, notably reduction or ablation of innate immune and T
cells, is necessary to prevent rejection of the HSCs in the
setting of allogeneic HSCT. Conditioning may also serve as
disease-directed therapy in allogeneic HSCT for malignant
disease. Serotherapy is a form of immunotherapy typically
involving antithymocyte globulin or alemtuzumab (monoclonal antibody to CD52) that is intended to address host
immune cell depletion, although it is primarily administered for in vivo GVHD prophylaxis. Total body irradiation
(TBI) is highly myelosuppressive but is associated with
many undesirable acute and late toxicities.
Myeloablative conditioning is standard for malignant
disease HSCT indications and has been used historically
for nonmalignant conditions as well. The goal of myeloablation is to replace all cell lines of the hematopoietic system (eg, lymphoid, myeloid) completely with donor HSCs.
Although most experts consider eradication of all recipient
blood cells to be essential for a person with leukemia, as few
as 20% donor cells in the decient cell line can reverse the
abnormal phenotype in a nonmalignant condition. (2) The
ability to cure a nonmalignant disease in the setting of
mixed-chimerism following RIC has greatly increased the
safety and application of HSCT to a broader number of
nonmalignant diseases. Graft failure after RIC often results in
autologous recovery of the recipients original HSCs.
Autologous HSCT conditioning regimens are almost
exclusively composed of high-dose combinations of

chemotherapy with or without radiation therapy targeted


at the underlying disease (usually malignant). The goal is to
rescue the patient after otherwise intolerable doses of these
agents given to intensify therapy.

RISKS OF HSCT
HSCT is associated with numerous acute and long-term
toxicities. The conditioning, its intensity (myeloablative
versus RIC), preexisting comorbidities, prior chemotherapy
exposure, and the stem cell source all inuence the risks of
complications and TRM. Children and adolescents generally tolerate myeloablative conditioning better than adults,
but TRM rates are still typically 5% to 10%. RIC was initially
designed to offer HSCT to patients with comorbidities, so
TRM rates are inherently lower, as are rates of many
toxicities. HSCT adverse effects on growth, development,
and fertility are especially pertinent in children and adolescents (Table 3). (11)(12) A detailed discussion of these late
effects is beyond the scope of this article, but comprehensive
follow-up by general pediatricians and a team with expertise
in HSCT late effects care and surveillance is recommended.
Surveillance guidelines have been published by the Childrens Oncology Group and other research bodies. (11)(12)(13)

Infections/Immune Reconstitution
HSCT usually involves myelosuppression as well as functional impairment of adaptive immunity. (14) As mentioned
previously, neutrophil engraftment typically occurs 2 to 3
weeks after HSC infusion, which is an important milestone
for innate immune protection against bacteria and fungi.
Natural killer cell recovery usually is complete by 1 month
post-HSCT, offering additional protection against infection.
T-cell function is impaired by intent during periods of
prophylaxis or therapy for GVHD, and GVHD in itself is
a dysregulated immune state, with poor function and protection against infection. For those HSCT recipients who
can stop GVHD prophylaxis by 6 months post-HSCT,
lymphocyte class switching (producing immunoglobulin
[Ig]G after IgM production) can be seen between 6 and 8
months after HSC infusion.
Children must be monitored for opportunistic infections
after HSCT. Bacteremia and sepsis are frequent, particularly
during the neutropenic phase before engraftment. Fungal
infections are also a concern during neutropenic phases or
corticosteroid therapy. Respiratory viruses such as respiratory syncytial virus and adenovirus can be devastating in an
immunocompromised host. Primary infection or reactivation with cytomegalovirus and Epstein-Barr virus (EBV)
warrant preemptive surveillance and intervention based

Vol. 37 No. 4
Downloaded from http://pedsinreview.aappublications.org/ by guest on April 2, 2016

APRIL 2016

139

TABLE 3. Late Effects of Pediatric Hematopoietic

Stem Cell Transplantation (11)( 12)


Endocrine

Growth disturbance (including growth


hormone deciency)
Hypothyroidism
Thyroid nodules
Hypogonadism
Delayed or precocious puberty
Infertility
Obesity (including sarcopenic obesity)
Osteopenia/osteoporosis
Avascular necrosis
Metabolic syndrome

Ophthalmologic

Cataracts
Xerophthalmia

Auditory

Hearing loss

Neurologic

Neurocognitive impairment
Cerebrovascular disease

Pulmonary

Pulmonary brosis
Bronchiolitis obliterans with or without
organizing pneumonia (usually chronic
GVHD)

Cardiovascular

Congestive heart failure


Conduction abnormalities
Valvular disease

Renal

Chronic kidney disease


Hypertension
Proteinuria

Gastrointestinal

Hepatic siderosis
Focal nodular hyperplasia of liver
Esophageal strictures
GVHD of upper or lower tracts
Hepatic GVHD

Secondary malignancy

Acute myelogenous leukemia (almost


exclusive to autologous HSCT)
Posttransplant lymphoproliferative
disease (non-Hodgkin lymphoma)
Solid tumors (skin, brain, thyroid,
musculoskeletal, oral cavity, breast,
gynecologic)

Dental

Disordered tooth eruption


Increased risk of caries
Xerostomia

Psychosocial

Depressed mood
Anxiety
Posttraumatic stress disorder
Relationship difculties
Vocational difculties
Chronic fatigue

GVHDgraft-versus-host disease, HSCThematopoietic stem cell


transplantation

on international guidelines and institutional practices. EBV


can be associated with posttransplant lymphoproliferative
disorder. Acyclovir prophylaxis for herpes simplex virus-1
(HSV-1) in seropositive recipients is generally administered

140

for up to 1 year post-HSCT and may also confer some


protection against varicella-zoster virus. (15)(16) BK virus
is a polyoma virus that is generally harmless in an immunocompetent host. However, it can cause hemorrhagic
cystitis and renal dysfunction in HSCT recipients if viremia
is present. Pneumocystis jiroveci prophylaxis is also indicated
until immune suppression has been withdrawn.
The Centers for Disease Control and Prevention, in
collaboration with several international HSCT organizations,
have established guidelines for infectious prophylaxis, and
international guidelines also exist for the management of
fever and neutropenia in pediatric HSCT recipients. (16)(17)
Finally, children require reimmunization after HSCT, but
clinicians must exercise caution regarding the timing of live
vaccine administration. Recommendations for the timing of
immunizations for children who have undergone HSCTcan
be referenced and are updated regularly. (16)(18)

Mucositis
Almost all children who undergo myeloablative HSCT
experience mucositis. This painful inammation of the
gastrointestinal mucosa is due to direct toxicity from conditioning agents and is compounded by a local inammatory state in the setting of neutropenia. It can occur
anywhere between the oral mucosa and rectum, and intensive intervention with narcotic and adjuvant therapies is
often required, with resolution typically occurring after
neutrophil engraftment. As the intensity of the conditioning
is reduced, the severity of mucositis decreases. Nutrition
support is commonly required while mucositis is present.
The risk of bacterial translocation across the lining of the
mucosa and secondary HSV-1 and fungal infections are a
concern.

Nutritional Support
Many children require nutritional supplementation postHSCT due to decreased intake, which may be related to
nausea, anorexia, malabsorption, or mucositis. Even when
many other complications abate, many children and adolescents need support to ensure adequate hydration and
caloric intake. In addition, metabolic needs are often
increased due to a catabolic state, with extensive tissue
healing required postconditioning. HSCT centers often
have strong preferences regarding the safest and most
benecial method of nutritional supplementation. Intractable
nausea and gut integrity, with potential compromise due to
mucositis or GVHD, should be considered when deliberating about enteral feeding. In the absence of contraindications, enteral feeding has potential benets to the liver in
promoting biliary ow, which is important because the liver

Pediatrics in Review
Downloaded from http://pedsinreview.aappublications.org/ by guest on April 2, 2016

is at risk of toxicity from conditioning, sinusoidal obstructive syndrome (SOS), GVHD, and polypharmacy.

Transfusions
Transfusion support is expected pre-engraftment, particularly for more intensive conditioning regimens. Optimization of iron load pre- and post-HSCT may reduce
complications (such as SOS), and phlebotomy post-HSCT
is employed for patients with high iron burdens post-HSCT
once stable engraftment has occurred. New research is
exploring the role of iron burden on inammation after
HSCT. (11)

Sinusoidal Obstructive Syndrome


SOS is a serious hepatoxicity seen in 1% to 10% of HSCT
recipients, with risk increased for those with preexisting
liver disease, allogeneic HSCT recipients, children with
high-risk neuroblastoma, and those who receive busulfan,
cyclophosphamide, or TBI. (19) It involves occlusion of
sinusoidal venules due to microthrombi, with resulting liver
swelling and enlargement, pain, uid retention, and
hepatorenal syndrome. Cholestasis is present, with variable
degrees of hepatic enzyme elevation. Although prevention
is ideal, treatment can vary from diuresis in moderate cases
to promising agents such as debrotide in severe cases
with end-organ failure. (20) Debrotide has yet to receive
approval from the US Food and Drug Administration.

agents. Although some degree of GVHD can be associated


with better overall survival for those with malignancy (due to
a graft-versus-leukemia effect), there is no benet in nonmalignant disease, and GVHD remains a barrier to the
application of HSCT for many nonmalignant conditions,
particularly if no matched family donor exists. (24)
Chronic GVHD is often seen months after HSCT (average onset at 6 months) and can be a devastating complication. (25) Clinically this entity resembles systemic lupus
erythematosus (SLE) or systemic sclerosis and can result in
debilitating skin, muscle, joint, liver, gut, and lung disease.
Dry eyes, dry mouth, and fatigue can also affect quality of
life. Prolonged immune suppression is required for more
severe cases, which can result in opportunistic infection.
(25) End-organ dysfunction, particularly of lung and liver, is
a major concern. Chronic GVHD can replace the condition
for which HSCT was performed, and although less common
in children than in adults, must always be considered during
the decision-making process for HSCT.
Tolerance between the donor T cells and the HSCT
recipient eventually results in the ability to reduce and
usually discontinue immune-suppressive medications.
Tapering of immune suppression occurs at scheduled time
points after HSC infusion in the absence of GVHD, with
longer periods of prophylaxis and higher target levels for
nonmalignant HSCTdisease indications. (26)( 27) For those
who develop GVHD, once the GVHD is inactive for a
sufcient period of time, immune suppression is weaned

Pulmonary Complications
The differential diagnosis of pulmonary complications after
HSCT is broad, with common causes being infection and
volume overload. Other considerations include pneumonitis from radiation or alkylating agents, idiopathic pneumonia syndrome, and chronic GVHD. Respiratory failure
requiring intubation and ventilation is associated with significant rates of mortality in immunocompromised recipients of HSCT. (21)

Graft-Versus-Host Disease
GVHD is an immune-based complication seen almost
exclusively in allogeneic HSCT. It involves tissue damage
and antigen exposure, antigen presentation, and alloimmune reactivity of donor T cells against recipient tissues.
(22) Acute GVHD affects the skin, gastrointestinal tract
(typically colon, stomach, or duodenum), and liver. Notably,
these organs are prone to chemotherapy and radiation
damage and are rich in antigen-presenting cells. Staging
systems describe the severity of each affected organ, with
an overall grading assigned. (23) Grading determines the
need to treat with corticosteroids and potentially additional

TABLE 4.

Malignant Disease Indications for


Allogeneic Pediatric HSCT

Acute lymphoblastic leukemia


Very high-risk features
Relapsed disease
Acute myelogenous leukemia
High-risk features
Relapsed disease
Myelodysplastic syndrome (preleukemic state)
Non-Hodgkin lymphoma
Relapsed/primary refractory disease
Disease subtype may indicate allogeneic versus autologous HSCT
Hodgkin lymphoma
Relapsed disease after autologous HSCT or primary refractory
disease (usually second relapse) (28)
HSCThematopoietic stem cell transplantation

Vol. 37 No. 4
Downloaded from http://pedsinreview.aappublications.org/ by guest on April 2, 2016

APRIL 2016

141

TABLE 5.

Nonmalignant Disease Indications for Allogeneic Pediatric


HSCT

Primary Immune Deciencies


Phenotype must be severe enough to justify HSCT
Specic genetic mutation identication ideal (can support indication for HSCT as well as inuence conditioning)
Hemoglobinopathies
Thalassemia major
Matched sibling or unrelated live donor
Unrelated UCB and haploidentical HSCT experimental
Sickle cell disease (Hg SS, S0, or SC)
Matched sibling donor
Unrelated donor and haploidentical HSCT experimental
Indications vary among centers, often some evidence of prior sickle cell complications required
Inherited Bone Marrow Failure Syndromes
Severe aplastic anemia
Fanconi anemia
Shwachman-Bodian-Diamond syndrome
Diamond-Blackfan anemia
Dyskeratosis congenita
Amegakaryocytic thrombocytopenia

Metabolic/Genetic Disorders (29)


Infantile osteopetrosis
Mucopolysaccharidoses
Hurler syndrome (MPS IH), standard of care
Optional indications (after frontline enzyme replacement therapy, if available)
n

Hurler/Scheie (MPS IH/S)

Scheie (MPS IS)

Maroteaux-Lamy (MPS VI)

Sly (MPS VII)

Leukodystrophies
Cerebral X-linked adrenoleukodystrophy
n

Before advanced disease

Metachromatic leukodystrophy, late onset


Krabbe disease, generally early onset
Miscellaneous disorders, optional indications
Fucosidosis
a-mannosidosis
Continued

142

Pediatrics in Review
Downloaded from http://pedsinreview.aappublications.org/ by guest on April 2, 2016

TABLE 5. (Continued )
Aspartylglucosaminuria
Farber
Gaucher types 1 (non-neuronopathic) and 3 (Norrbottnian)
Niemann-Pick type C-2
Wolman syndrome
HSCThematopoietic stem cell transplantation, MPSmucopolysaccharidosis, UCBumbilical cord blood

and subsequently stopped. For this reason, HSCT recipients


are not expected to receive lifelong immune suppression, in
contrast to patients who receive solid organ transplantation.

INDICATIONS FOR HSCT


Historically, most allogeneic HSCT procedures in children
were for malignant diseases such as leukemias and lymphomas. With improving cure rates using chemotherapy for
such cancers, the proportion of nonmalignant disease indications for pediatric HSCT continues to increase.

Malignant Disease
Common malignant disease indications for allogeneic
HSCT in children are acute leukemias and some nonHodgkin and Hodgkin lymphomas. High-risk clinical/
biological features or relapse are usually present (Table
4). Myelodysplastic syndrome, a preleukemic state with risk
of conversion to acute myeloid leukemia, is almost always
treated with HSCT in children. Chronic myelogenous leukemia is often managed with tyrosine kinase inhibitors
alone, so fewer affected children and adolescents are recommended to undergo HSCT.
Autologous HSCT is performed routinely for children
with high-risk neuroblastoma and for relapsed lymphomas.
Many brain tumor treatment plans are incorporating highdose chemotherapy and autologous HSCT, particularly for
children younger than age 3 years, in an effort to spare or
delay radiation therapy to the developing brain. Current
research is exploring the use of autologous HSCT in children and adolescents with solid tumors, such as Ewing
sarcoma, who have high-risk features.

Nonmalignant Disease
Allogeneic HSCT is increasingly performed for nonmalignant disease indications as rates of TRM and GVHD are
reduced. These diseases confer lifelong risks of morbidity or
mortality and often require complex supportive care (Table 5).

Chronic transfusions for hemoglobinopathies are associated with signicant risks of iron overload and resultant
complications. For some of these conditions, the risks of
HSCT are affected substantially by the type of donor available, and the resulting recommendation for HSCT may be
affected. Primary immune deciencies such as severe combined immune deciency, X-linked chronic granulomatous
disease, and Wiskott-Aldrich syndrome are examples of
nonmalignant diseases for which HSCT is commonly performed. A large body of evidence supports the safety and
efcacy of HSCT for severe aplastic anemia, with increasing
data to guide clinicians in decision-making for inherited
bone marrow failure syndromes. Thalassemia major has an
established track record for related and unrelated HSCT,
with a clear phenotype of lifelong transfusion dependence
and risk of iron overload.
Sickle cell disease (SCD) is increasingly recognized as a
disease with limited life expectancy and variable quality of life
despite best supportive care. As a result, interest is growing in
the application of HSCT to those with sickling syndromes.
Although a history of complications of SCD had been mandated in the past to justify HSCT, the safer HSCT techniques
have prompted increasing interest from patients, hematologists, and HSCT practitioners to intervene before organ
dysfunction occurs, notably neurologic and lung injury.
Some metabolic diseases such as mucopolysaccharidoses are routine indications for HSCT, although the potential benets are less clear for other metabolic diseases. (29)
Table 5 summarizes some of the more standard indications,
with an acknowledgement that HSCT is performed in some
centers for life-threatening metabolic diseases with fewer
data regarding potential benet. (29) HSCTcan help prevent
neurologic progression in a metabolic disease due to
replacement of the decient enzyme by monocytes produced from the HSCs following engraftment. Because
HSCT generally only halts and does not reverse neurologic
progression and knowing that HSC-derived enzyme
replacement can take months to reach the central nervous

Vol. 37 No. 4
Downloaded from http://pedsinreview.aappublications.org/ by guest on April 2, 2016

APRIL 2016

143

system (CNS) due to slow migration of donor-derived


monocytes into the CNS, early HSCT is critical for indicated
conditions. Generally other non-CNS manifestations of the
metabolic disorders are not reversed with HSCT. Discussions about the appropriateness of HSCT should happen
relatively soon after making a diagnosis, and those who
manage such conditions routinely should be aware of
evolving indications for this group of diseases.
The practice of autologous HSCT for nonmalignant
conditions is relatively limited. Some encouraging results
for those with severe SLE suggest that some patients may
derive temporary benet in terms of corticosteroid-sparing
or reduced disease activity. (30) Repopulation of the bone
marrow and peripheral blood with fewer autoreactive
clones, in addition to the use of disease-modifying agents
such as cyclophosphamide as part of the conditioning, may
explain this period of improvement. Gene therapy trials for
hemoglobinopathies are incorporating autologous HSC collection and ex vivo manipulation, with reinfusion following
conditioning designed to give the manipulated cells a survival
advantage. (31) The use of autologous HSCT for traumatic
brain injuries and cerebral palsy is an area of intense research,
but these indications are experimental at present.

Summary
Hematopoietic stem cell transplantation (HSCT) refers to the
infusion of either allogeneic or autologous hematopoietic stem
cells.
Newer techniques to reduce the risk of complications are
expanding the applicability of HSCT.
Nonmalignant disease indications for HSCT are increasing.
Observational and cohort studies (level C evidence) indicate that
acute and long-term toxicities remain an important consideration
for patients, families, and clinicians in making a recommendation
for HSCT and warrant lifelong surveillance. (11)(12)(13)(21)
Based on overwhelming evidence from observational studies
(level B evidence), graft-versus-host disease can be a signicant
cause of morbidity and mortality in allogeneic HSCT. (22)(24)
General pediatricians and subspecialists should be aware of
evolving and newly established nonmalignant indications for
HSCT to make appropriate referrals (level D evidence). (28)(29)
(30)

CME quiz and references for this article are at http://pedsinreview.


aappublications.org/content/37/4/135.

Parent Resources from the AAP at HealthyChildren.org


https://www.healthychildren.org/English/health-issues/conditions/cancer/Pages/Cancer-Therapies.aspx
Spanish: https://www.healthychildren.org/Spanish/health-issues/conditions/cancer/Paginas/Cancer-Therapies.aspx

144

Pediatrics in Review
Downloaded from http://pedsinreview.aappublications.org/ by guest on April 2, 2016

PIR Quiz
There are two ways to access the journal CME quizzes:
1. Individual CME quizzes are available via a handy blue CME link under the article title in the Table of Contents of any issue.
2. To access all CME articles, click Journal CME from Gateways orange main menu or go directly to: http://www.aappublications.
org/content/journal-cme.

REQUIREMENTS: Learners
can take Pediatrics in
Review quizzes and claim
credit online only at:
http://pedsinreview.org.

1. A 13-year-old girl with acute myeloblastic leukemia has relapsed 6 months after
completing her initial course of chemotherapy. You explain to the parents that the only
potential cure will be hematopoietic stem cell transplantation (HSCT). Which of the
following options is the best donor for this girl?
A.
B.
C.
D.
E.

Allogenic transplant using


Allogenic transplant using
Allogenic transplant using
Allogenic transplant using
Autologous transplant.

a rst cousin who matches at 8/10 loci.


a sibling who matches at 8/10 loci.
an unrelated donor who matches at 8/10 loci.
her mother who matches at 8/10 loci.

To successfully complete
2016 Pediatrics in Review
articles for AMA PRA
Category 1 CreditTM,
learners must
demonstrate a minimum
performance level of 60%
or higher on this
assessment, which
measures achievement of
the educational purpose
and/or objectives of this
activity. If you score less
than 60% on the
assessment, you will be
given additional
opportunities to answer
questions until an overall
60% or greater score is
achieved.

2. Which of the following would be the best therapy for the child described in the previous
question?
A.
B.
C.
D.
E.

Chemotherapy alone to attempt prolonged remission.


Myeloablative conditioning prior to transplant.
Reduced-intensity conditioning prior to transplant.
Serotherapy prior to transplant.
Total body irradiation prior to transplant.

3. A 5-year-old boy underwent HSCT 12 days ago because of neuroblastoma. He is now


complaining of increasing abdominal pain. You note that he has icterus and mild
generalized edema. Laboratory studies reveal a total bilirubin of 4.5 mg/dL (76.9 mmol/L)
and conjugated bilirubin of 2 mg/dL (34.2 mmol/L) but only mild elevations in
transaminase values. The most likely cause of his symptoms is:
A.
B.
C.
D.
E.

Cytomegalovirus.
Hepatitis A.
Hepatitis B.
Sepsis.
Sinusoidal obstructive syndrome.

4. A 7-year-old girl with homozygous sickle cell anemia underwent HSCT from an unrelated,
human leukocyte antigen-identical donor 7 months ago. She has been complaining of
fatigue for 2 weeks and now has developed a feeling of her mouth being dry. On physical
examination she has a widespread nonspecic erythematous rash over her trunk and arms.
There is no cyanosis or jaundice. She has shotty anterior cervical nodes but no other
signicant adenopathy. The most likely cause of her symptoms is:
A.
B.
C.
D.
E.

This journal-based CME


activity is available
through Dec. 31, 2018,
however, credit will be
recorded in the year in
which the learner
completes the quiz.

Acute graft-versus-host disease.


Chronic graft-versus-host disease.
Cytomegalovirus.
Epstein-Barr virus.
Human herpesvirus 6.

5. A 4-year-old girl presents with bruising and pallor. She is found to have pancytopenia. A
bone marrow aspirate and biopsy are diagnostic of myelodysplastic syndrome. Which of
the following is the most appropriate treatment for this childs myelodysplastic syndrome?
A.
B.
C.
D.
E.

Begin chemotherapy and evaluate the response long term.


Begin prophylactic antibiotics to prevent sepsis.
Maintain the patient on transfusions until she becomes unresponsive to them.
Observe the child until the pancytopenia becomes severe.
Proceed to HSCT once an appropriate donor is identied.

Vol. 37 No. 4
Downloaded from http://pedsinreview.aappublications.org/ by guest on April 2, 2016

APRIL 2016

145

Hematopoietic Stem Cell Transplantation in Children and Adolescents


Gregory M.T. Guilcher
Pediatrics in Review 2016;37;135
DOI: 10.1542/pir.2015-0044

Updated Information &


Services

including high resolution figures, can be found at:


http://pedsinreview.aappublications.org/content/37/4/135

References

This article cites 26 articles, 7 of which you can access for free at:
http://pedsinreview.aappublications.org/content/37/4/135#BIBL

Permissions & Licensing

Information about reproducing this article in parts (figures, tables) or


in its entirety can be found online at:
http://beta.pedsinreview.aappublications.org/site/misc/Permissions.x
html

Reprints

Information about ordering reprints can be found online:


http://beta.pedsinreview.aappublications.org/site/misc/reprints.xhtml

Downloaded from http://pedsinreview.aappublications.org/ by guest on April 2, 2016

Hematopoietic Stem Cell Transplantation in Children and Adolescents


Gregory M.T. Guilcher
Pediatrics in Review 2016;37;135
DOI: 10.1542/pir.2015-0044

The online version of this article, along with updated information and services, is
located on the World Wide Web at:
http://pedsinreview.aappublications.org/content/37/4/135

Pediatrics in Review is the official journal of the American Academy of Pediatrics. A monthly
publication, it has been published continuously since 1979. Pediatrics in Review is owned,
published, and trademarked by the American Academy of Pediatrics, 141 Northwest Point
Boulevard, Elk Grove Village, Illinois, 60007. Copyright 2016 by the American Academy of
Pediatrics. All rights reserved. Print ISSN: 0191-9601.

Downloaded from http://pedsinreview.aappublications.org/ by guest on April 2, 2016

Physical Abuse of Children


Jill C. Glick, MD,* Michele A. Lorand, MD, Kristen R. Bilka, MMS, PA-C
*Department of Pediatrics, University of Chicago; Medical Director, Child Advocacy and Protective Services, University of Chicago Comer Childrens Hospital,
Chicago, IL.

Division of Child Protective Services, Department of Pediatrics; Medical Director, Chicago Childrens Advocacy Center, John H. Stroger, Jr. Hospital of Cook
County, Chicago, IL.

Department of Pediatrics, University of Chicago; Physician Assistant, Child Advocacy and Protective Services, University of Chicago Comer Childrens Hospital, Chicago, IL.

EDITORS NOTE
This article stresses the importance of the sentinel injury, a physical injury that
is unusual for the age of the child and may herald more serious injuries, thereby
necessitating further evaluation.
Joseph A. Zenel, MD
Editor-in-Chief

Practice Gap
Before receiving a diagnosis of child abuse, 25% to 30% of abused infants
have sentinel injuries, such as facial bruising, noted by clinicians or
caregivers. (1)(2)(3)(4)(5)(6) Although easily overlooked and often considered
minor, such injuries are harbingers warning clinicians that pediatric patients
require further assessment. Appropriate intervention is critical, and the
clinician plays a major role in identifying children who present with signs
or symptoms concerning for child physical abuse by ensuring appropriate
and expeditious medical evaluations and reports to child protective services.

Objectives

After completing this article, the reader should be able to:

1. Identify which injured children require a child abuse evaluation.


2. Recognize subtle signs and nonspecic symptoms of major trauma in
infants.
3. Understand sentinel injuries and their signicance.
4. Know which laboratory and imaging studies to obtain when child
physical abuse is suspected.
5. Understand the legal obligation to report children with injuries that are
suspicious for physical abuse and develop a thoughtful approach to
informing parents of this legal obligation.

CASE PRESENTATION
A private practice pediatrician receives a phone call from a community emergency
department (ED) physician regarding one of her patients, a 4-month-old infant
being treated for bronchiolitis. The ED physician informs her that the babys chest

146

AUTHOR DISCLOSURE Drs Glick and Lorand


and Ms Bilka have disclosed no nancial
relationships relevant to this article. This
commentary does not contain a discussion of
an unapproved/investigative use of a
commercial product/device.

Pediatrics in Review
Downloaded from http://pedsinreview.aappublications.org/ by guest on April 2, 2016

radiograph has revealed multiple posterior rib fractures in


different stages of healing, and physical examination shows
a cluster of small bruises on her cheek. The mother denies a
history of trauma and has no explanation for the ndings.
The ED physician is concerned that the baby has been
abused and his plan includes admitting the patient to the
hospital to obtain a head computed tomography (CT) scan,
skeletal survey, complete blood cell count, coagulation studies, electrolytes, and liver function tests. He also plans to
consult with the child abuse pediatrician and arrange for an
evaluation of the patients siblings. Lastly, he tells the
primary pediatrician that he will explain the clinical ndings
to the family and le a report with the child welfare system.
The primary pediatrician thanks him for contacting her and,
recalling no signicant medical history, pulls the patients
chart.
The babys most recent visit was slightly more than 1
week ago for her routine 4-month health supervision
visit. She is a term infant who has no prior medical
complaints other than colic at 1 month of age that has
resolved. On recent physical examination, the baby appeared well, with normal growth and development, and
the mother did not raise any concerns during the visit.
The primary pediatrician now notes that she documented
a small circular bruise on the babys chest that the mother
stated occurred when a 3-year-old sibling hit the baby with a
toy. Having had a longstanding relationship with this
mother and family, she accepted this explanation for the
bruise.
After reviewing the chart, she explores the current
literature and management of suspected child physical
abuse, including the American Academy of Pediatrics
clinical report on evaluation of suspected child physical
abuse. (7) She now understands that the bruise she noted
on examination was a sentinel injury that should have
prompted further evaluation. As a result of the case, her
practice group plans to review and implement guidelines
for the identication and evaluation of children presenting with signs or symptoms concerning for physical
abuse.

Clinicians must appreciate that with few exceptions, almost


any injury can be either abusive or accidental.
Once considered a strictly social problem, child abuse is
now also recognized as a medical problem. A recent survey
by the Childrens Hospital Association revealed that more
than 90% of responding hospitals have child protection
teams, and more than 50% have at least 1 of the 324 boardcertied child abuse pediatricians in the United States on
staff. (8)
Recognition of the profound impact of childhood experiences on adult health and well-being, beginning with
Feleittis landmark adverse childhood experiences study,
further solidies the need for clinicians to recognize
possible maltreatment and intervene. (9) Adverse childhood experiences have wide-ranging, cumulative, and
direct impacts on adult health, increasing the incidence
of chronic diseases and early death. (9)(10) The role of
the clinician is therefore not only limited to promoting
wellness but also to decreasing or eliminating long-term
health consequences resulting from childhood exposure to
trauma and violence.

EPIDEMIOLOGY
In 2014, over 3.5 million children were subjects of child
maltreatment reports. Of those, 702,000 children (20%)
were found to have evidence of maltreatment. (11) This
translates to an annual victimization rate of 9.4 children
per 1,000 in the United States and a prevalence rate of 1 in
8 children by age 18 years. (12) Neglect is the most
common form of child maltreatment, constituting 75%
of indicated reports; 7% are attributable to physical abuse.
In 80% of child physical abuse cases, a biological parent is
the perpetrator. Children in their rst postnatal year have
the highest victimization rate (24.4 per 1,000), and children younger than age 3 years have the highest fatality
rate, comprising over 70% of the nationally estimated
1,580 child maltreatment deaths in 2014. Child welfare
data and trends, however, are dubious because of a lack of
standardized terminology and differences in report and
response types across states.

INTRODUCTION
Child physical abuse is a difcult diagnosis to entertain
primarily because clinicians are hesitant to accept that
caretakers can injure children. The diagnosis is further
complicated by the reality that caretakers rarely disclose
maltreatment, preverbal or obtunded children cannot provide
a history, and signs and symptoms of physical abuse may be
subtle and confused with other common pediatric diagnoses.

RISK FACTORS FOR CHILD PHYSICAL ABUSE


Risk factors for abuse are commonly categorized into
parental, child, and social characteristics. Identication of
risk factors aids in the assessment of abuse but more
importantly aids in the ability to counsel parents and
develop preventive strategies. Risk factors are not, in
and of themselves, diagnostic. Many families have risk

Vol. 37 No. 4
Downloaded from http://pedsinreview.aappublications.org/ by guest on April 2, 2016

APRIL 2016

147

factors and never abuse their children, while others have


no apparent risk factors and do abuse their children. Child
abuse does not discriminate; it affects children of all ages,
socioeconomic classes, and ethnic groups.
Parental/household risk factors include substance
abuse, mental illness, interpersonal violence (IPV), single
and/or teen parent, and a nonrelated adult in the home.
Among the social risk factors are social isolation, poverty,
lower levels of education, and large family size. Childrelated risk factors include prematurity, low birthweight,
intrauterine drug exposure, and developmental and physical disabilities. The most signicant risk factor for abuse is
the age of the child, with infants and toddlers being at
greatest risk for serious and fatal child physical abuse.
A clear association exists between particular developmental stages and physically abusive injuries, such as excessive
crying and abusive head trauma or toilet training and
inicted scald burns. Awareness of these developmental
triggers should guide anticipatory guidance, with the potential for preventing an abusive injury.
IPV is a substantial risk factor for child abuse, and each
health supervision visit should include IPV screening.
Exposure to violence itself, even if the child is not physically
harmed, has signicant and long-lasting effects.

WHEN TO CONSIDER THE DIAGNOSIS OF CHILD


PHYSICAL ABUSE
Injuries are common in childhood. Although most childhood injuries are accidental, the clinician must appreciate
that almost any injury can be abusive. With the exception of
patterned marks, very few injuries are pathognomonic for
abuse. In the nonverbal child, injuries may be apparent or
covert; many children present with nonspecic symptoms
and a lack of history. Child physical abuse should be
entertained in any infant displaying signs or symptoms
potentially explained by trauma, such as irritability, lethargy,
vomiting, apnea, seizures, or coma.
Several studies of abused children have demonstrated
that antecedent sentinel injuries, such as bruises, intraoral
lesions, and skeletal trauma, were noted by medical professionals or caregivers before a subsequent abusive act,
while children presenting with accidental injuries were not
found to have sentinel injuries. (1)(2)(3)(4)(5)(6) Because
infants are essentially nonmobile and nonweight-bearing,
they should never have bruising. Therefore, any injury in
an infant must be viewed as signicant and descriptive
language such as minor should not be used. Identifying
a sentinel injury with appropriate evaluation of the child may
be lifesaving.

148

As children become mobile, the incidence of expected


accidental trauma increases, and common childhood
injuries such as bruises over bony prominences and
toddlers, clavicular, and skull fractures are seen. In
contrast to children with abusive injury, witnesses often
corroborate accidental injuries in ambulatory children,
caregivers seek timely care, they provide a consistent
history, and the mechanism described explains the injury
observed. Because the incidence of child physical abuse is
highest in children younger than age 4 years, the clinician
must have a high index of suspicion and add abusive
trauma to the differential diagnosis of the ill-appearing
young child.
Determining which injured children require an evaluation for child physical abuse should account for the age and
developmental ability of the child, the injury sustained, the
adequacy of the historical explanation provided, and

TABLE 1.

Criteria for Consideration to Initiate a


Child Physical Abuse Assessment

Age and Development


Nonmobile infant with any injury
Injury in nonverbal child
Injury inconsistent with childs ability
Statement of harm from a verbal child
Injury
Any injury in a nonmobile infant
Uncommon in age group
Occult nding
Mechanism not plausible
Multiple injuries, including involvement of multiple organs
Injuries of differing ages
Pattern of increasing frequency or severity of injury over time
Patterned cutaneous lesions
Bruises to torso, ear, or neck in child younger than age 4 years
Burns to genitalia, stocking or glove distribution, branding, or pattern
History
Chief complaint does not contain caregiver concern for an injury
and plausible history
Caretaker response not commensurate to injury
Unexplained delay in seeking care
Lack of, inconsistent, or changing history
Inconsistencies or discrepancies in histories provided by
involved caretakers

Pediatrics in Review
Downloaded from http://pedsinreview.aappublications.org/ by guest on April 2, 2016

clinical ndings (Table 1). Fundamentally, when injuries


are not explained or historical data provided contain inconsistencies or insufciencies, a child abuse evaluation is
warranted. Any child younger than age 2 years who presents with a suspicious injury should have a skeletal
survey. Other studies should be obtained based upon
clinical concern and ndings. Negative studies do not rule
out child abuse.

HISTORY OF THE PRESENT ILLNESS AND CHILD


PHYSICAL ABUSE
A thorough history of present illness is the single most
useful piece of information to aid the clinician in making a
correct diagnosis. The detailed history should be obtained
in separate interviews with each caregiver, the child (if
possible), verbal siblings, and any other persons in the
household. Interviews should be conducted such that each
parent or caregiver can give a history in his or her own
words. He or she should be allowed to provide the entire
history without interruption, decreasing the chance that
the interviewer unintentionally redirects or suggests a
mechanism. Details about the mechanism of injury, the
events leading up to the injury, and whether the injury was
witnessed or unwitnessed should be elicited. For example,
in injuries related to falls, having parents recreate the
scene, describing the height of furniture, ooring, and
the position of the child before and after the fall, is
essential.
A history of the onset and progression of symptoms
since the child last appeared well should be obtained.
Determining who was caring for the child and asking each
of the caretakers how the child appeared by focusing on
descriptions of activity and movement (particularly during
feeding, bathing, and diaper changing) can aid in determining when a child may have been injured. For infants
with intracranial injury, it may be difcult to develop a
timeline of when the child was last well because the infant
may be thought of as well-appearing while asleep when
the child actually may be seriously injured. Important
features of the history that should raise concern for an
abusive injury include: no history of trauma; a history of
trauma inconsistent with the severity, pattern, or timing of
the injury; injury inconsistent with the developmental
capabilities of the child; multiple or evolving histories;
discrepant histories from the same caregiver or between
caregivers; injury attributed to a sibling or pet; and a delay
in seeking medical care.
In addition to a detailed history of the incident, the
patients birth, past medical, developmental, and dietary

histories should be obtained. A complete social history


identies risk factors for maltreatment, and a family medical history focusing on illnesses such as bone disease or
bleeding tendencies allows for screening and identication
of possible underlying medical problems in the patient.

PHYSICAL EXAMINATION AND DIAGNOSTIC


EVALUATION
A thorough and well-documented physical examination of
any child with concerns for possible child abuse is imperative. The clinician should be aware that children may suffer
more than one type of abuse; the physically abused child
may also be neglected or sexually abused. The childs mental
status, affect, and level of activity should be noted. The child
must be undressed and all skin surfaces examined with
good lighting. The entire body must be evaluated, including
areas that may be overlooked, such as the pinnae, behind the
ears, the oral cavity including the teeth and frenula, the soles
and palms, the genitals, and the anus. Every cutaneous
injury should be described according to color, shape, size,
and location. Photographic documentation or drawings
should be completed and placed in the medical record.
The presence or absence of swelling and the ability to move
limbs should be noted. Paradoxical comfort (a baby who is
more comfortable when not being held but cries when
picked up) may be observed in infants with occult injuries
such as rib fractures. An assessment of the childs nutritional status, including completion of a growth chart, is
crucial because neglect, malnutrition, and failure to thrive
may be comorbidities with physical abuse.
The diagnostic evaluation of suspected physical abuse
should always be driven by the history, physical examination,
and differential diagnosis. Clinicians must consider the possibility that multiple types of trauma may coexist and recognize
that injuries may be occult. Any nonverbal and nonambulatory
child with an injury should have a standard child abuse
evaluation (Table 2) no matter how minor the injury. The
most prudent approach is to rule out skeletal trauma in all
children younger than 2 years of age with a standard skeletal
survey and assess for occult central and/or internal injuries
by choosing appropriate imaging and laboratory studies
(Table 3).

ABUSIVE HEAD TRAUMA


Abusive head trauma (AHT) has the highest mortality of
all forms of child physical abuse, with an estimated
fatality rate greater than 20%. Survivors have irreversible
sequelae of brain injury, ranging from minor behavioral

Vol. 37 No. 4
Downloaded from http://pedsinreview.aappublications.org/ by guest on April 2, 2016

APRIL 2016

149

TABLE 2.

Protocol for the Evaluation of Suspected Child Physical Abuse

History of Present Illness


Interview primary caretakers separately; note historians ability to provide history
Ask caretakers about age-appropriate developmental abilities of child. Observe child if possible
Develop a timeline from when the child was last agreed upon to be in his or her usual state of good health and note the following:
B

Onset of symptoms and progression

The patients observed mental status and activity level. Ask specically about how the child appeared at time of hand off between
caretakers

Note if there were any witnesses, photos taken of child, or other corroborating information
Social History
List all adults having access to the child, including age, relationship, and contact information
List all children, including age and relationship; identify in which home they reside
Note history of drug or alcohol abuse, intimate partner violence, mental illness, prior history of involvement with child protective services
Relevant Past Medical History
Skeletal trauma: child or family history of bone disease, diet history
Abusive head trauma (AHT) and cutaneous injuries: child or family history of bleeding diathesis, eg, prolonged bleeding after circumcision,
umbilical cord removal, or surgery or as a result of past injuries
Physical Examination
Examine closely for possible intraoral injuries such as frenulum tears; explore all unexposed surfaces: behind ears, genital region, and bottoms of feet
Growth chart: obtain prior growth data, and with regard to AHT, note trajectory of head circumferences
Photodocumentation
If photos are obtained, document in the medical record details of the photos taken, including location of injuries, number of photos taken,
date, and photographer
If photodocumentation is unavailable, use a body diagram noting all cutaneous lesions by size, location, and color
Evaluation
Indicated laboratory and imaging studies for current illness or injury
Studies to assess occult injuries, such as skeletal survey
Communication with appropriate subspecialists regarding ndings and treatment, including child abuse pediatricians when appropriate for
referral and consultation
Mandated Reporting and Safety
Develop dialogue to inform parents about mandated reporting, safety, and reason for report
Ensure that forms and phone numbers for reporting are accessible
Establish ofce process for specic scenarios with regard to obtaining imaging and laboratory studies and process for transfer to appropriate
facility for evaluation and treatment, including protocol for accessing expertise of child abuse pediatrician
Facilitate thorough sibling assessment, including appropriate imaging, laboratory studies, and interpretation; establish protocol to ensure
results of sibling assessments are communicated to others in the investigation, including primary care clinician
Ensure medical record and photodocumentation accessibility for investigators (consent not required after report to child welfare)
Discuss disposition of, medical follow-up, and supportive services for patient with child welfare case worker

issues and neurodevelopmental delays to signicant neurodevelopmental delays, seizures, blindness, and paralysis. (13) The incidence of AHT is 15 to 30 cases per
100,000 infants annually in the United States. AHT

150

occurs most often in children younger than age 2 years


and crying is the most commonly identied trigger.
Recognizing that the phrase shaken baby syndrome
implies a specic mechanism, in 2009 the American

Pediatrics in Review
Downloaded from http://pedsinreview.aappublications.org/ by guest on April 2, 2016

Academy of Pediatrics (AAP) recommended that AHT


replace this terminology to acknowledge that multiple
mechanisms, either separately or together, can cause
calvarial, brain, and cervical injuries. (14)
Infants and young children who have AHT can present
with signs and symptoms ranging from mild to lifethreatening, with a clinical spectrum that includes irritability, vomiting, lethargy, seizures, apnea, coma, and death.
Often there are no external ndings suggestive of trauma
and the history is lacking or misleading. Thus, depending on
the extent and severity of the injuries, traumatic brain injury
is often misdiagnosed as colic, viral syndrome, otitis media,
gastroenteritis, gastroesophageal reux, or pyloric stenosis.
Clinicians must keep AHT in their differential diagnoses
and have a high index of suspicion to obtain a thorough
history and perform the appropriate diagnostic tests.
Brain injuries seen in 80% of AHT cases include subdural hemorrhage that is interhemispheric, posterior,
often layering over the tentorium, and/or a thin subdural
layer over either or both of the convexities. Mass effect
results not from the subdural trauma itself but rather from
signicant cerebral edema. The parenchymal damage
evolves into a clinical picture consistent with hypoxicischemic encephalopathy. Although additional injuries
need not be present to diagnose AHT, these neurologic
injuries are frequently associated with other traumatic
ndings, such as retinal hemorrhages, posterior rib fractures, and classic metaphyseal lesions (CMLs). Bruising to
the scalp or other parts of the body may or may not be present.
The clinician must be mindful to ensure a thorough evaluation for other occult injuries, including neck, internal,
and other skeletal trauma.
A short fall leading to fatal head trauma is exceptionally
rare, with a calculated risk of less than 1 per 1,000,000
children annually. (15) A unique situation is the development of an epidural hematoma after minor blunt trauma in
which a temporal linear skull fracture may sever the middle
meningeal vessels and lead to an accumulation of blood that
results in mass effect. This is one circumstance in which a
child may be neurologically intact after minor or trivial
trauma but experience deteriorating mental status and acute
symptoms as a result of mass effect.
Head and neck imaging must be obtained for any child
for whom there are concerns for AHT. A CT scan is the
initial imaging modality of choice because it can be performed quickly in the critically ill child. However, CT scan
does not reveal parenchymal injuries, cannot reliably differentiate between subdural and subarachnoid collections,
and involves substantial radiation exposure. Magnetic resonance imaging (MRI) should be performed once the

patient is stable, ideally a few days after admission, to


optimize visualization of the parenchyma and evaluate for
edema, stroke, and thromboses. MRI can also elucidate the
location of extra-axial uid collections and aid in the aging of
intracranial hemorrhages. MRI imaging of the spine is also
indicated because studies have now demonstrated injury to
the cervical spine, such as ligamentous injury and spinal
subdural hemorrhage, in children with AHT. (16)(17) MRI is
also preferred over CT scan when clinical ndings such as
rapidly increasing head circumference or focal neurologic
issues suggest remote injury. MRI does not entail radiation
exposure, but it is a longer study that most often requires
sedation.
Retinal injuries such as hemorrhages, schisis or tearing,
and folds are associated with AHT and may be seen in up
to 80% of cases. Hemorrhagic retinopathy from AHT is
classically described as multilayered, with hemorrhages
that are too numerous to count and extend to the ora
serrata. This very specic nding is unique to AHT and
is not due to increased intracranial pressure, blunt head
trauma, or cardiopulmonary resuscitation. Retinoschisis
and macular folds are reported almost exclusively in children who have sustained violent craniorotational injury
and are specic to this mechanism. Any infant or child who
has intracranial injuries suspicious for abuse should be
evaluated by an ophthalmologist who can meticulously and
precisely document the ocular ndings, preferably with
use of photo imaging. The rate of healing varies from days
to weeks and aging of retinal hemorrhages is imprecise. Of
note, an ophthalmologic examination is not a screening
tool for AHT but is indicated when there is evidence of
intracranial injury.

CUTANEOUS INJURIES
The skin is the most frequently injured organ in child
abuse, with bruises, bites, and burns accounting for many
child maltreatment injuries. Although cutaneous injuries
are very common in childhood, they are rare in the
preambulatory child: those who dont cruise dont
bruise. (18)(19) Considerable data support that bruising
is not only extremely uncommon in infants but highly
correlated with child abuse. (20)(21) Thirty percent or
more of seriously injured or fatally abused children have
been noted to have bruises, which are sentinel signs
(Figure 1), reported on physical examination before subsequent severe or fatal abuse. These data support the
directive that any nonmobile infant who has a bruise
must receive a full child abuse evaluation (Table 2) and a
report to child welfare for investigation. (3)(4)(5)(6)

Vol. 37 No. 4
Downloaded from http://pedsinreview.aappublications.org/ by guest on April 2, 2016

APRIL 2016

151

TABLE 3.

Child Physical Abuse Medical Evaluation: Imaging and


Laboratory Studies

STANDARD CHILD PHYSICAL ABUSE MEDICAL EVALUATIONS


Skeletal Injuries
Skeletal survey (with views according to the collaborative practice parameter issued by the American College of Radiology and the Society for
Pediatric Radiology)
Follow-up skeletal survey is indicated in 2 weeks when abuse is suspected on clinical grounds and/or initial ndings are abnormal or
equivocal
Core laboratory studies for bone health: calcium, magnesium, phosphate, and alkaline phosphatase
If concerns for vitamin D deciency (elevated alkaline phosphate, abnormal bone density, or dietary concerns), consider 25-hydroxyvitamin D
and parathyroid hormone level
Central Imaging
Head computed tomography (CT) scan (useful for screening, and/or monitoring an ill child)
Magnetic resonance imaging (MRI) of head and spine (useful for elucidating extra-axial spaces, parenchymal disease, and spinal injury)
Routine Trauma Laboratory Tests
Hematologic: complete blood cell count and platelets
Coagulation: international normalized ratio, prothrombin time, and activated partial thromboplastin time
Metabolic: glucose, blood urea nitrogen, creatinine, calcium, magnesium, phosphate, albumin, and protein
Urinalysis: urine toxicology screen, order myoglobin if urinalysis positive for blood and red cells are not seen on smear
Liver function tests: aspartate aminotransferase and alanine aminotransferase (>80 U/L [1.34 mkat/L] is concerning for occult injury)
Pancreatic enzymes: amylase and lipase
ADDITIONAL POTENTIAL TESTS
Ophthalmologic Examination
Indicated if evidence of either acute or remote central nervous system trauma
Not a screening tool for abusive head trauma
Abdominal (Thoracoabdominal) Imaging: CT Scan With Intravenous Contrast
Elevated liver or pancreatic enzyme values
Comatose patient
Evidence of trauma with delay in care (liver function tests may have decreased to normal levels)
Concerns for Bleeding Diathesis (Family History or Clinical Concerns)
von Willebrand antigen, von Willebrand activity (ristocetin co-factor), Factor VIII, Factor IX, platelet function assay
Hematology consultation
Metabolic Diseases
Genetics consultation

As children start to ambulate, the incidence of bruising


increases. Bruise location and morphology are important
factors to consider when assessing for child physical abuse
in ambulatory children. Accidental injuries tend to occur
over bony prominences (shins and elbows) in contrast to
bruises due to abuse, which are located on the face, head,
neck, torso, anks, buttocks, and thighs. The mnemonic

152

TEN 4 is useful to recall which bruise locations are


concerning for abuse: Torso, Ear, Neck, and 4 signifying
children younger than age 4 years and any bruising noted in
infants younger than 4 months. (20) Bruising and abrasions
that occur on more than one body surface, are in multiple
stages of healing, and are patterned or well demarcated are
more likely to be the result of abuse. Patterned injuries

Pediatrics in Review
Downloaded from http://pedsinreview.aappublications.org/ by guest on April 2, 2016

reect the shape of the instrument, such as loop marks


from a cord or cable, linear bruises from belts, or multiple
parallel linear bruises equally distributed from a slap with a
hand. Contrary to some common beliefs, children do not
bruise more easily than adults and bruises cannot be aged
precisely. The appearance of a bruise is related to many
factors, including the state of hemoglobin degradation, the
color of skin pigment, the depth of the bruise, the location
on the body, the lighting in the room, and the patients
metabolism and circulation. Bruises of differing colors do
not signify different times or incidents. Finally, because
bruises or soft-tissue injury may be painful for days, the
presence of tenderness does not necessarily mean the
injury is acute.
Determining whether marks or bruising from corporal
punishment constitutes abuse is a difcult task. Legally,
some states condone corporal punishment as an acceptable
form of behavioral modication while others dene it as a
form of child maltreatment and require reporting to child
welfare. The AAP and the American Academy of Child and
Adolescent Psychiatry do not condone corporal punishment due to its limited effectiveness and potential deleterious effects. Although known to be immediately effective,
spanking and corporal punishments have signicant
adverse outcomes, such as increased aggression and
decreased development of appropriate behavior. (22)(23)
(24) Both groups advise against the use of corporal punishment and encourage alternative methods of behavioral
modication such as time out, loss of privileges, positive
reinforcement, and opportunities for positive touch like
hand holding and hugging that have healthier, long-lasting
effects. From a practical standpoint, each clinician must be
versed in his or her state laws. More importantly, the
clinician must develop a thoughtful and culturally sensitive

Figure 1. Infant displaying bruising that represents a sentinel sign for


physical abuse.

dialogue with parents that promotes alternative methods of


discipline.
Bite marks are another patterned skin injury noted in
abused children. Clinicians can discern between animal and
human bites by assessing the shape: animal bite marks are
puncture wounds with a sharply angulated arch, while
human bite marks are crush injuries consisting of an ovoid
pattern of tooth marks that may surround an area of central
bruising. In general, adult bite marks measure greater than
2 cm between the maxillary canines. Consultation with a
forensic odontologist may assist in the evaluation of welldemarcated bite marks. Multiple bites on different body
planes, bites on soft-tissue areas, and bites on areas generally covered by clothing should raise a suspicion of abuse.
Bites to the genitalia, buttocks, and/or breasts should raise a
concern of possible sexual abuse. Acute bites to the genitalia,
buttocks, and/or breasts may warrant collection of forensic
evidence for DNA by swabbing the area with a cotton swab
moistened with distilled water.
Hot liquid, grease, steam, hot objects, chemicals, electricity, or microwave ovens may cause abusive burns. Compared to accidental burns, abusive burns are more severe,
more likely to be full-thickness, and require more extensive
treatment, including grafting. Children who are abusively
burned are most often younger than 4 years and inicted
immersion burns to the buttocks and genitalia are commonly associated with toilet training.
Abusive burns most often take the form of immersion
scald burns, characterized by well-demarcated areas of
conuent depth with no splash or cascading ow pattern.
Immersion burns may involve the buttocks, perineum,
extremities, hands, or feet. Circumferential burns affecting the feet and/or hands are sometimes referred to as
having a stocking or glove distribution. In immersion
burns, the position in which the child was held may be
surmised by the burn pattern and depth. If the childs
buttocks come into contact with the tub surface, a doughnut type pattern may be noted with relative sparing of the
part of the anatomy coming into contact with the tub. Sparing
of the exion creases is often observed. Persons who inict
these burns generally do not suffer burns themselves. A
careful scene reconstruction and investigation, including
water temperature, may help determine the length of time
the child was held in the water. Generally, the hotter the water,
the shorter the duration of submersion. Partial-thickness
burns develop in minutes at 48.9C (120F) but take mere
seconds at 65.6C (150F). (25)
Children often come into contact with hot objects, such
as irons, hair tools, radiators, and stovetops. Resultant burns
are related to the heat of the object and period of contact with

Vol. 37 No. 4
Downloaded from http://pedsinreview.aappublications.org/ by guest on April 2, 2016

APRIL 2016

153

the skin. Both abusive and accidental contact burns can


result in a patterned mark, making discernment based on
appearance difcult. Suspicious contact burns require a
thorough scene investigation and corroboration, particularly
if the child is nonverbal. In general, abusive contact burns
are deeper and leave a clear imprint while those that result
from grazing against a hot object are not as deep or welldemarcated.
The most common differential diagnosis of inicted
cutaneous injury is accidental injury. Dermatologic and
other conditions such as congenital dermal melanocytosis (mongolian spots), phytophotodermatitis, HenochSchnlein purpura, Ehlers-Danlos syndrome, vasculitis
syndromes, bleeding dyscrasias, eczema, malignancy, and
cultural practices such as cupping and coining (cao gio or
gua sha) may lead to cutaneous lesions that appear initially
concerning for abuse. Some conditions that may be confused with or mimic burns are impetigo, staphylococcal
scalded skin syndrome, herpes, and contact dermatitis. A
careful history, physical examination, and diagnostic tests
should clarify whether an accident or one of these conditions is the cause of the ndings. As always, clinicians
should be mindful that the child who has an underlying
medical problem might also be physically abused.

SKELETAL INJURIES
Fractures are the second most common type of child
physical abuse. Accidental fractures are common in ambulatory children but not in nonambulatory children. Most
abusive fractures occur in nonambulatory children, representing 55% to 70% of fractures in children younger than
age 1 year and 80% of all abuse fractures found in children
younger than 18 months of age. (26) Age is the single most
important risk factor for abusive skeletal injury.
Understanding that different types of fractures result
from different forces applied to the bone aids in determining if a given history is plausible. Transverse fractures are
due to forces that are perpendicular to the bone or bend the
bone, torus or buckle fractures are due to axial loading or
compression along the long axis of the bone, spiral fractures
are due to twisting, and oblique fractures are due to a
combination of transverse and twisting forces.
Abusive skeletal injury may involve any part of the
skeleton, but fractures of the extremities are most common.
Any fracture can result from abuse, and no fracture is
pathognomonic for abuse. Some fractures, however, have
a higher specicity for abuse, such as posterior or lateral
rib fractures or CMLs, also known as corner or bucket
handle fractures. These fractures occur at the ends of long

154

bones, resulting from twisting that causes a planar fracture


through the spongiosum of the metaphysis.
Some uncommon fractures, such as sternal, spinal, and
scapular fractures, are also highly suggestive of child abuse
in the absence of a credible and corroborated trauma history.
Any child who presents with multiple fractures or fractures
in differing stages of healing should raise concern for
nonaccidental trauma.
The most common differential diagnosis of abusive
fracture is accidental fracture. Some fractures, such as
clavicular fractures, may be due to birth trauma. Underlying
medical conditions and metabolic bone diseases should be
considered in the differential diagnosis of skeletal trauma
and include osteogenesis imperfecta, Menkes syndrome,
hyperparathyroidism, hypophosphatasia, and Fanconi syndrome. Although vitamin D deciency is prevalent, rickets
is not, and research demonstrates that the incidence of
fractures in skeletal trauma is not inuenced by vitamin
D deciency. (27)(28)

IMAGING OF SKELETAL INJURIES


Skeletal injuries may be clinically silent in children younger
than age 2 years and in developmentally delayed or nonverbal children. A skeletal survey to identify acute or healing
fractures should be performed when there is concern for
any form of child physical abuse. If an abused child has
siblings who are younger than 2 years, skeletal surveys
should be performed to evaluate these siblings. The current
recommendation by the American College of Radiology for
skeletal survey consists of 21 dedicated views, including
oblique views of the chest to aid in the detection of rib
fractures. (29) Although the clinician must be mindful of
radiation exposure, the discovery of an occult injury is a
major determinant in the diagnosis of child abuse and, thus,
outweighs radiation risk. Infants may have positive skeletal
surveys in up to 20% of cases. (30)
Some fractures with high specicity for child abuse, such
as rib fractures and CMLs, may not be apparent on the initial
skeletal survey. With healing and new bone formation, these
injuries become more obvious. Thus, a repeat skeletal survey
obtained 10 to 14 days after the initial survey is recommended
to re-evaluate for fractures. Bone scans are no longer obtained
as a complementary test to detect possible injury because of
their serious limitations, including problems with motion
artifact, inability to estimate age of injury, and lack of specicity. Due to radiation exposure concerns, CT scan is not a
preferred imaging modality to detect fractures. If chest or
abdominal CT scan is performed for other reasons, it can be
useful in detecting nondisplaced rib fractures. Skull fractures

Pediatrics in Review
Downloaded from http://pedsinreview.aappublications.org/ by guest on April 2, 2016

in the line of the axial plane are often missed on head CTscan.
The addition of 3-dimensional CT scan reconstruction
enhances the identication and morphology of skull fractures
and helps ensure their detection and diagnosis.
Aging of bone fractures is imprecise and must be based
upon history and clinical examination ndings as well as
radiographic known patterns of healing. In young children
with long bone fractures, new bone is visible within 1 to 2
weeks, followed by callous development, disappearance of
the fracture line, and nally resolution. Some fractures, such
as skull fractures or CMLs, do not follow a predictable healing
process and cannot be aged based on radiographs alone.

CHEST AND ABDOMINAL INJURIES


Abdominal trauma is the second leading cause of fatalities
due to child physical abuse. This is likely due to delays in
seeking medical care, a misleading history of no or only
trivial trauma, and the greater severity of injury in abusive as
compared to accidental abdominal trauma. Early signs and
symptoms, such as loss of appetite, vomiting, and abdominal pain, are nonspecic and may be misdiagnosed. Furthermore, physical examination generally does not reveal
bruising to the chest or abdomen. Some children with
abdominal trauma may be battered and have distracting
injuries, such as AHT or a fracture, that may delay recognition of the abdominal injury. Because signs and symptoms of chest and abdominal injuries may be subtle or
overlooked, meticulous physical examination is imperative.
Signs and symptoms of occult abdominal trauma vary,
depending on the age of child and presence of associated
injuries such as intra-abdominal hemorrhage or peritonitis.
The clinician should palpate the chest wall to examine for rib
deformities. Chest wall tenderness and/or palpable callus
may suggest the presence of healing rib fractures. Auscultation for equal breath sounds as well as clear heart tones
and bowel sounds and palpation of the abdomen must be
performed. Because the physical examination results can be
misleading, with a lower sensitivity for trauma in younger
children, laboratory screening should be considered, particularly in any nonverbal or nonambulatory child.
Relative to accidental abdominal trauma, abusive abdominal trauma occurs more often in infants and toddlers, is
more severe, and requires a higher rate of surgical intervention. The peak age for abusive abdominal injury is
between ages 2 and 3 years. This may be due to child-related
behavioral risk factors, such as increased activity level,
normal exploration, normal negativism, and toilet training.
Abusive injuries to almost every visceral structure have
been reported. The most commonly injured organs are the

liver and spleen, followed by duodenal and proximal jejunal


ruptures or hematomas and pancreatic, vascular, and/or
renal trauma. Other injuries may involve the bladder, large
intestine, stomach, mesentery, and adrenals. The most common mechanism for abdominal injury is focused blunt force
trauma to the abdomen that compresses, crushes, or tears
the viscera. Because the forces are focused internally, abdominal bruising is rare, and clinicians should not be lulled into a
false sense of security by the absence of abdominal bruising.
The major diagnostic differential diagnosis for abusive
abdominal injury is accidental abdominal injury, which is
readily elucidated by history (or lack thereof).
Any child in whom abdominal trauma is suspected
should undergo screening, including liver function tests,
pancreatic enzymes, hemoglobin to assess for blood loss,
and urinalysis to check for hematuria. Plain radiographs are
rarely diagnostic but may reveal indirect evidence of visceral
injury, such as dilated loops of bowel, air uid levels, free air,
bowel wall air, periportal tracking, or mass effect on the
bowel. Abdominal CT scan with contrast is the preferred
imaging modality to assess for intra-abdominal injury and
is indicated when hepatic enzymes aspartate aminotransferase and alanine aminotransferase measure more than
80 U/L (1.34 mkat/L). (31) Liver enzyme values acutely rise
but then rapidly fall after blunt trauma; liver enzymes due to
infection or liver disease do not resolve in this pattern. In
children who have subacute abdominal trauma, liver enzymes
may have normalized by the time of evaluation. (32)
Thoracic injuries are mostly related to crush or major
blunt trauma and may involve the heart, lungs, rib cage, and
mediastinum. Pulmonary hemorrhage and edema due to
airway obstruction, shearing injury to the thoracic duct
resulting in a chylothorax, pneumomediastinum, pneumopericardium, and commotio cordis have all been described
as a result of abusive trauma.

REPORTING TO CHILD WELFARE AND THE ROLE OF THE


CLINICIAN
As mandated reporters, clinicians are required to make a
report to child welfare when there is reasonable cause to
believe that an injury is the result of abuse or neglect. The
clinician need only suspect that maltreatment has occurred
to initiate a report. The clinicians role is to initiate evaluation and ensure appropriate treatment and safety of the
patient. The nal determination of child abuse requires a
coordinated interdisciplinary investigation. Access to the
expertise of a child abuse specialist is ideal because he or she
is accustomed to collaborating with law enforcement, child
welfare, and the legal system.

Vol. 37 No. 4
Downloaded from http://pedsinreview.aappublications.org/ by guest on April 2, 2016

APRIL 2016

155

Primary care clinicians may be challenged by their


responsibility to inform the caregiver or parent(s) of a report
suspicious for child maltreatment. They are understandably
concerned about the safety and disposition of their young
patients as well as the potential stress and reaction of the
family to an investigation. Anticipating a negative family
reaction may cause hesitation in reporting. However, the
clinician must keep in mind his or her role as the childs
advocate and recognize that the report is a medical intervention that may be lifesaving. Developing a thoughtful
dialogue to inform a parent of the need to make a
report can ease the stress on the reporter as well as the
parent.
Clinicians often wonder what happens after a report is
made. In every state, a child welfare organization is
responsible for investigation of the child maltreatment
allegation. The investigative agency may substantiate
(indicate) or not substantiate (unfound) a particular allegation. Unsubstantiated ndings do not necessarily mean
that child maltreatment did not occur. Most investigations
do not result in removal of a child from a home and may, in
fact, provide opportunities for augmenting the family by
offering support to caregivers, such as home visits, parenting classes, access to transportation for follow-up care,
and individual or group therapy.
Some cases may progress in the legal system. When there
is urgent and immediate concern for a childs safety, the case
may be heard in juvenile/family court and result in the
temporary removal of a child to ensure his or her safety,
health, and well-being. These children are assigned their
own attorney, known as a guardian ad litem.
Criminal court testimony is requested when a specic
person has been charged with an act leading to an abusive
injury. Unlike juvenile court, in which evidence is based
upon preponderance (more likely than not), criminal court
requires evidence beyond a reasonable doubt. Managing
Child Abuse: General Principles is an excellent reference
(33) that provides a very thoughtful stepwise approach to
reporting and navigating the subsequent legal process.
No matter which direction a report may take, the clinician
must advocate for his or her patient to have necessary
resources and an appropriate medical home.

signicant contemporary source of morbidity and mortality. Appreciation of the adverse impacts of early stressors on adult health has been transformational in
validating the role of the pediatric clinician in promoting
wellness not only in childhood but also into adulthood.
Knowledge of child physical abuse continues to evolve,
providing more clarity, as demonstrated by the recent
understanding of the signicance of sentinel injuries.
Increased awareness of child maltreatment by primary
care clinicians along with timely intervention ideally can
lead to effective prevention of the adverse outcomes of
child maltreatment and, along with more dedicated research,
to effective primary prevention.

Summary
On the basis of research and consensus, the diagnosis of child
physical abuse must be entertained whenever an infant or
nonverbal child presents with any injury. Substantial evidence
supports that any form of trauma in a baby is signicant and
deserves complete evaluation. (1)(2)(3)(4)(5)(6)
Clinicians must consider child abuse in the differential diagnosis
of any young child with injuries or symptoms where there are
discrepancies between the sustained injuries and the history and/
or patients developmental capabilities. On the basis of strong
research and consensus, child abuse is recognized not only as a
major source of mortality and morbidity in childhood but also as a
direct cause of increased adult morbidity and early death. (9)(10)
On the basis of consensus, primary care clinicians are in a position
to identify children with injuries concerning for child abuse, initiate
an appropriate and thoughtful medical evaluation, report to child
welfare, and appropriately seek child abuse pediatric consultation.

CME quiz and references for this article are at http://pedsinreview.aappublications.org/content/37/4/146.

To view PowerPoint slides that accompany this article,


visit http://pedsinreview.aappublications.org
and click on the Supplemental tab for this article.

PREVENTION AND FUTURE HORIZONS


The monetary cost to society of child abuse has been
estimated to be $80.3 billion per year. (34) However, this
does not adequately reect the true loss of a child due to
untimely and preventable death. Child maltreatment is
now recognized as a major public health problem and a

156

Pediatrics in Review
Downloaded from http://pedsinreview.aappublications.org/ by guest on April 2, 2016

PIR Quiz
There are two ways to access the journal CME quizzes:
1. Individual CME quizzes are available via a handy blue CME link under the article title in the Table of Contents of any issue.
2. To access all CME articles, click Journal CME from Gateways orange main menu or go directly to: http://www.
aappublications.org/content/journal-cme.

REQUIREMENTS: Learners
can take Pediatrics in
Review quizzes and claim
credit online only at:
http://pedsinreview.org.

1. A 14-month-old girl is brought to the emergency department with history of a fall from a
couch to a tile oor. Her mother and father have accompanied her. The mother believes
the fall was about 2.5 feet. She reports that the girl cried immediately and after a few
minutes, she seemed to act in a typical manner. She also reports that after about 15
minutes the girl started vomiting and was sleepy. You obtain a computed tomography
scan of the brain that shows a 9-mm right-sided epidural hematoma with mass effect.
What is the most appropriate next step in management?
A.
B.
C.
D.
E.

To successfully complete
2016 Pediatrics in Review
articles for AMA PRA
Category 1 CreditTM,
learners must
demonstrate a minimum
performance level of 60%
or higher on this
assessment, which
measures achievement of
the educational purpose
and/or objectives of this
activity. If you score less
than 60% on the
assessment, you will be
given additional
opportunities to answer
questions until an overall
60% or greater score is
achieved.

Obtain social work consultation.


Request consultation from the child abuse specialist.
Request magnetic resonance imaging (MRI) of the brain.
Request a neurosurgical consultation.
Separate the parents and obtain histories from each of them.

2. You see a 3-month-old boy for a health supervision visit. His mother reports that he spits up
after most feedings. He has eczema over his face, arms, and chest. He has a nickel-sized
bruise behind his left ear. His mother reports that he rolled onto a toy in his crib and this
caused the bruise. What is the most appropriate next step in management?
A.
B.
C.
D.
E.

Obtain complete blood cell count and iron levels.


Request MRI of the brain.
Request referral for a dermatology evaluation.
Request referral to a gastroenterologist.
Submit a report to child welfare.

3. A 1-year-old boy has bruising over his back and upper arms with several parallel lines of
bruising on both upper arms. Several of the bruises are green; other bruises are yellow and
purple. You are asked to testify in court regarding the cause and timing of these injuries.
You testify that:
A.
B.
C.
D.
E.

Children bruise more easily than adults.


The parallel lines of bruising likely represent a hand print injury.
These sites of bruising are common in accidental injuries.
Varied colors of bruising show that there were injuries at varied times.
You are unable to provide any information related to the boys bruises.

This journal-based CME


activity is available
through Dec. 31, 2018,
however, credit will be
recorded in the year in
which the learner
completes the quiz.

4. You see a 2-year-old boy with vomiting and weight loss over the past several weeks. His
father relates that several other family members have had gastrointestinal illness in the
past month. On physical examination, the boys abdomen is mildly distended with bilateral
upper quadrant tenderness. He appears mildly dehydrated and drinks small amounts of
water during the visit without emesis. He wants to be held and cries throughout the
examination. What is the most appropriate next step in management?
A.
B.
C.
D.
E.

Obtain complete blood cell count, liver transaminases, and pancreatic enzymes.
Prescribe antacid daily.
Request MRI of the brain.
Request referral for ophthalmology evaluation.
Request referral to a gastroenterologist.

5. You see a 3-year-old girl for a respiratory illness. Her mother states that the girl refuses to
go to sleep and wakes multiple times during the night. The girl is resistant to toilet training
and her mother reports that she holds her on the toilet to help her potty train. She has had
a few successful voids on the toilet with this method. She has several bruises on her right
scapula that her mother reports occurred when she fell from a dining room chair. She has a
faint bruise on her right facial cheek and her mother is not sure of how this injury occurred.
Her physical examination ndings are otherwise normal. What is the most appropriate next
step in management?

Vol. 37 No. 4
Downloaded from http://pedsinreview.aappublications.org/ by guest on April 2, 2016

APRIL 2016

157

A.
B.
C.
D.
E.

Obtain laboratory studies for possible bleeding disorder.


Request polysomnography evaluation of her sleep.
Request renal ultrasonography and cystometrography.
Request referral for ophthalmology evaluation.
Submit a report to child welfare.

Parent Resources from the AAP at HealthyChildren.org


https://www.healthychildren.org/English/safety-prevention/at-home/Pages/What-to-Know-about-Child-Abuse.aspx
Spanish: https://www.healthychildren.org/Spanish/safety-prevention/at-home/Paginas/What-to-Know-about-Child-Abuse.aspx

Addendum for Meningitis


Regarding the December 2015 Pediatrics in Review article Meningitis (Pediatr Rev. 2015;36(12):514526, doi: 10.1542/
pir.36-12-514:
The incidence of Haemophilus inuenzae type b (Hib) invasive disease, including meningitis, has decreased
tremendously with the increased use of Hib conjugate vaccine in infants. Unfortunately, there is concern that similar
to other vaccine-preventable diseases, such as measles and pertussis, an upsurge in Hib meningitis could follow a
decrease in Hib vaccine use. Therefore, it is important for clinicians to recognize possible Hib meningitis promptly and
treat it effectively.
The footnote to the listing for Escherichia coli in Table 6 of the article on meningitis published in Pediatrics in Review
clearly states Or other Gram-negative enteric bacilli. Choice of antibiotic is directed by the results of susceptibility
testing. What may not be clear to all readers is that a very small percentage of Hib that are beta-lactamase-negative still
have a sufciently high minimum inhibitory concentration (MIC) for ampicillin to make Hib resistant to ampicillin.
Thus, ampicillin is not considered a preferable antibiotic until susceptibility (based on MIC) is available. This is an
essential point because the consequences of initial ineffective therapy can be disastrous.
Out of an abundance of caution, we want to remind readers (and have updated the online version of the article with
the notation) that The Committee on Infectious Diseases of the American Academy of Pediatrics recommends, Initial
therapy for children with H inuenzae meningitis is cefotaxime or ceftriaxone. Ampicillin should be substituted if the
Hib isolate is susceptible.
Mobeen H. Rathore, MD, FAAP
Pediatrics in Review Editorial Board
AAP Committee on Infectious Diseases

158

Pediatrics in Review
Downloaded from http://pedsinreview.aappublications.org/ by guest on April 2, 2016

Physical Abuse of Children


Jill C. Glick, Michele A. Lorand and Kristen R. Bilka
Pediatrics in Review 2016;37;146
DOI: 10.1542/pir.2015-0012

Updated Information &


Services

including high resolution figures, can be found at:


http://pedsinreview.aappublications.org/content/37/4/146

References

This article cites 29 articles, 15 of which you can access for free at:
http://pedsinreview.aappublications.org/content/37/4/146#BIBL

Subspecialty Collections

This article, along with others on similar topics, appears in the


following collection(s):
Medical Education
http://beta.pedsinreview.aappublications.org/cgi/collection/medical_
education_sub
Journal CME
http://beta.pedsinreview.aappublications.org/cgi/collection/journal_c
me
Emergency Medicine
http://beta.pedsinreview.aappublications.org/cgi/collection/emergenc
y_medicine_sub
Trauma
http://beta.pedsinreview.aappublications.org/cgi/collection/trauma_s
ub
Injury, Violence & Poison Prevention
http://beta.pedsinreview.aappublications.org/cgi/collection/injury_vi
olence_-_poison_prevention_sub
Child Abuse and Neglect
http://beta.pedsinreview.aappublications.org/cgi/collection/child_abu
se_neglect_sub

Permissions & Licensing

Information about reproducing this article in parts (figures, tables) or


in its entirety can be found online at:
http://beta.pedsinreview.aappublications.org/site/misc/Permissions.x
html

Reprints

Information about ordering reprints can be found online:


http://beta.pedsinreview.aappublications.org/site/misc/reprints.xhtml

Downloaded from http://pedsinreview.aappublications.org/ by guest on April 2, 2016

Physical Abuse of Children


Jill C. Glick, Michele A. Lorand and Kristen R. Bilka
Pediatrics in Review 2016;37;146
DOI: 10.1542/pir.2015-0012

The online version of this article, along with updated information and services, is
located on the World Wide Web at:
http://pedsinreview.aappublications.org/content/37/4/146

Data Supplement at:


http://pedsinreview.aappublications.org/content/suppl/2016/03/29/37.4.146.DC1.html

Pediatrics in Review is the official journal of the American Academy of Pediatrics. A monthly
publication, it has been published continuously since 1979. Pediatrics in Review is owned,
published, and trademarked by the American Academy of Pediatrics, 141 Northwest Point
Boulevard, Elk Grove Village, Illinois, 60007. Copyright 2016 by the American Academy of
Pediatrics. All rights reserved. Print ISSN: 0191-9601.

Downloaded from http://pedsinreview.aappublications.org/ by guest on April 2, 2016

Diplopia in a 15-year-old Boy


Christine Puthawala, DO,* Shana Hansen, MD
*Submarine Base New London, Groton, CT.

Adolescent Clinic, Fort Sam Houston, TX.

PRESENTATION
EDITORS NOTE
We invite readers to contribute Index of
Suspicion cases at: Submit and Track My
Manuscript.
AUTHOR DISCLOSURE Drs Puthawala and
Hansen have disclosed no nancial
relationships relevant to this article. This
commentary does not contain a discussion of
an unapproved/investigative use of a
commercial product/device.

A 15-year-old previously healthy boy presents to the adolescent clinic with bilateral
frontal headaches over the last 4 months. He was initially treated with antibiotics
for presumed sinusitis. The headaches improved after 2 weeks of therapy,
decreasing in frequency to once a month. They are sometimes present upon
awakening but never wake him from sleep. At the time the headaches began, the
patient also developed progressively worsening double vision. He also reports
hearing his heartbeat in his left ear for several weeks starting 9 months ago. For
the last 10 days he has experienced impaired balance when his eyes are shut. He
denies travel, trauma, fevers, weight loss, mood changes, weakness, sensory
decits, vomiting, nausea, or a history of Lyme disease or meningitis. He has no
signicant past medical history or history of developmental delay.
On physical examination, his temperature is 36.7C (98.1F), heart rate is 95
beats per minute, respiratory rate is 16 breaths per minute, and blood pressure is
112/77 mm Hg. He is a healthy-appearing teenager in no acute distress. Mental
status is normal. Pupils are equally reactive, but bilateral optic disc edema is noted.
No nystagmus, ptosis, or visual decits are present. Cranial nerves are normal, with
the exception of an inability to abduct the right eye with horizontal movement,
which is exacerbated with gaze to the right. The teen also has binocular diplopia.
He has 3 patellar reex on the left and the rest of the deep tendon reexes are 2.
His strength is 5/5 in the upper and lower extremities, muscle tone is normal,
and sensation to light touch is intact. His gait, including tandem gait, is normal.
A Romberg test and further cerebellar testing yield normal results.
Laboratory evaluation is deferred for an emergency computed tomography
(CT) scan.

DISCUSSION
CT scan revealed hydrocephalus with bilaterally enlarged lateral ventricles and
third ventricle. Signs of hemorrhage, mass effect, and infarction were not present.
Magnetic resonance imaging (MRI) documented narrowing of the cerebral
aqueduct with right displacement of the pineal gland and dilation of the vein of
Galen. No vein of Galen aneurysm was noted. MRI was negative for any spaceoccupying lesions. The patient was diagnosed with aqueductal stenosis and
ultimately received a ventriculoperitoneal shunt, which resolved his hydrocephalus
and diplopia.

Vol. 37 No. 4
Downloaded from http://pedsinreview.aappublications.org/ by guest on April 2, 2016

APRIL 2016

169

The Condition
Late-onset idiopathic aqueductal stenosis (LIAS) is easily
missed because it may present without a history of trauma,
meningitis, or an intracranial hemorrhage. Although hydrocephalus is generally diagnosed in neonates and young
children, its incidence has a bimodal distribution, with the
most frequent diagnoses seen in infants younger than age
12 months and adolescents.
The pathophysiology of LIAS is not completely known.
The hypothesis is that the process begins with decreased
compliance of the intracranial veins, which results in
increased development of venous collateral ow. This
impairs brain compliance and results in aqueductal occlusion followed by ventriculomegaly. Pediatric patients, such
as the one presented, develop symptoms of increased intracranial pressure (ICP), while older adults develop ataxia,
cognitive impairment, and incontinence more typical of
normal-pressure hydrocephalus.
Of note, hydrocephalus can be stable for years. Such
arrested hydrocephalus can acutely decompensate after
head trauma, hemorrhage, and infections or after compromise of the compensatory capacities of the brain. Alternatively, hydrocephalus may develop more gradually as the
increased pressure places greater sheer stress on the aqueduct, which eventually causes symptomatic damage. In
addition, increased pressure in the supratentorial ventricles
may cause kinking of the aqueduct.
Signs of hydrocephalus include macrocephaly, visual
disturbances, papilledema, Collier sign (setting sun sign),
and Parinaud syndrome (upward gaze paralysis). If increased ICP is present, patients may have symptoms such
as headache, as seen in this patient. Pupillary dilation and a
paralysis of the light reex is a late nding. Ten percent of
adolescents with aqueductal stenosis exhibit symptoms
related to compression of the hypothalamic-hypophyseal
axis, including obesity, diabetes insipidus, precocious puberty,
and amenorrhea.

Differential Diagnosis
Diplopia has numerous causes. In many cases, the diagnosis can be reached with a detailed history and physical
examination.
The rst step is to determine whether the diplopia is
monocular or binocular. If the diplopia is only present with
both eyes open, the patient has binocular diplopia. The
clinician must determine the cause of ocular misalignment. Ascertaining whether the diplopia is horizontal,
vertical, or oblique is helpful. The most common causes
of diplopia generally involve extraocular muscle dysfunction, although binocular diplopia may also be due to eye

170

displacement. Monocular diplopia usually has an intraocular pathology, which necessitates expedited ophthalmologic
assessment.
Periorbital pain or pain with eye movements may
suggest an inammatory process. Ptosis and worsening
diplopia with intensive use of the eye may indicate
myasthenia gravis. Weakness of proximal limb muscles
may be due to mitochondrial myopathy or congenital
myopathy. Progressive headache and papilledema suggest increased ICP.
The differential diagnosis for hydrocephalus and subsequent increased ICP is extensive, with traumatic brain
injury resulting in cytotoxic edema being the most common cause in the pediatric population. Increased ICP is
also commonly caused by infections, such as bacterial
meningitis, mumps, cytomegalovirus, inuenza A, and
in the fetus, toxoplasmosis. Hemorrhage within the ventricles or brain matter can also lead to elevated ICP and may
result in hydrocephalus if debris obstructs the ow of
cerebrospinal uid. Space-occupying lesions, including
tumors, aneurysms of the vein of Galen or basilar artery,
and arteriovenous malformations, can cause increased
ICP. Among the additional causes of elevated ICP are
genetic diseases such as the X-linked L1 syndrome
(Bickers-Adams-Edwards syndrome) and neurobromatosis type-1. Central nervous system malformations such as
Chiari and Dandy-Walker malformations or spina bida
also may result in hydrocephalus. Finally, hydrocephalus
may be caused by functional stenosis that causes large
pressure differences between the supra- and infratentorial
spaces.
There are several mechanisms by which the previously
listed causes trigger aqueductal stenosis and, thereby,
hydrocephalus. For example, an external mass may simply
force the aqueduct shut. Intrinsic pathology of the aqueduct may cause an occlusion by stenosis of the aqueduct,
forking, septum formation, or gliosis following inammation from infection, toxins, or hemorrhage. Experimental
evidence also suggests that certain toxins (trypan blue,
salicylates, and cuprizone) and dietary deciencies (vitamin A, vitamin B, and folic acid) can cause stenosis of the
cerebral aqueduct.

Management and Prognosis


The management of hydrocephalus is highly dependent on
the cause. All cases warrant MRI. Corticosteroids, furosemide, and acetazolamide have limited success in treating
acute hydrocephalus. No data support the concept that
medications decrease the number of patients eventually
requiring ventriculoperitoneal shunting.

Pediatrics in Review
Downloaded from http://pedsinreview.aappublications.org/ by guest on April 2, 2016

One of the most common treatments for hydrocephalus is


ventriculoperitoneal shunting. Although the mortality rate
associated with the procedure is low, 40% of shunts fail within
12 months, after which the failure rate stabilizes at 5% annually.
Half of failures are due to obstruction, but other complications
include infection (most commonly Staphylococcus epidermidis in
the rst 6 months), subdural hematomas due to overshunting,
headaches, and nausea. Craniosynostosis is a potential complication of ventriculoperitoneal shunting specic to infants.
Prognosis depends on the cause of hydrocephalus, the
time from onset until diagnosis, and the effectiveness of
treatments. Hydrocephalus caused by genetic conditions has
the worst prognosis. For nontumor-associated hydrocephalus,
as seen with this patient, the 10-year mortality rate is between
5% and 15%.

Lessons for the Clinician


Hydrocephalus can present in the pediatric population
without a history of trauma, infection, or tumor. The
presentation may include visual and gait disturbances and
headache.
Imaging with computed tomography scan or magnetic
resonance imaging is recommended for cases of suspected elevated intracranial pressure.
Approved for public release; distribution is unlimited. The
views expressed herein are those of the authors and do not
necessarily reect the ofcial position of the Department of
Defense or its components.
Suggested readings for this article are at http://pedsinreview.
aappublications.org/content/37/4/169.

ANSWER KEY FOR APRIL 2016 PEDIATRICS IN REVIEW


Hematopoietic Stem Cell Transplantation in Children and Adolescents: 1. B; 2. B; 3. E; 4. B; 5. E.
Physical Abuse of Children: 1. D; 2. E; 3. B; 4. A; 5. E.
Syncope: 1. C; 2. A; 3. A; 4. D; 5. A.

Vol. 37 No. 4
Downloaded from http://pedsinreview.aappublications.org/ by guest on April 2, 2016

APRIL 2016

171

Case 1: Diplopia in a 15-year-old Boy


Christine Puthawala and Shana Hansen
Pediatrics in Review 2016;37;169
DOI: 10.1542/pir.2015-0148

Updated Information &


Services

including high resolution figures, can be found at:


http://pedsinreview.aappublications.org/content/37/4/169

References

This article cites 3 articles, 1 of which you can access for free at:
http://pedsinreview.aappublications.org/content/37/4/169#BIBL

Permissions & Licensing

Information about reproducing this article in parts (figures, tables) or


in its entirety can be found online at:
http://beta.pedsinreview.aappublications.org/site/misc/Permissions.x
html

Reprints

Information about ordering reprints can be found online:


http://beta.pedsinreview.aappublications.org/site/misc/reprints.xhtml

Downloaded from http://pedsinreview.aappublications.org/ by guest on April 2, 2016

Case 1: Diplopia in a 15-year-old Boy


Christine Puthawala and Shana Hansen
Pediatrics in Review 2016;37;169
DOI: 10.1542/pir.2015-0148

The online version of this article, along with updated information and services, is
located on the World Wide Web at:
http://pedsinreview.aappublications.org/content/37/4/169

Pediatrics in Review is the official journal of the American Academy of Pediatrics. A monthly
publication, it has been published continuously since 1979. Pediatrics in Review is owned,
published, and trademarked by the American Academy of Pediatrics, 141 Northwest Point
Boulevard, Elk Grove Village, Illinois, 60007. Copyright 2016 by the American Academy of
Pediatrics. All rights reserved. Print ISSN: 0191-9601.

Downloaded from http://pedsinreview.aappublications.org/ by guest on April 2, 2016

Acute Onset of Lower Extremity Weakness in


a 16-year-old Korean Boy

Sandy Aikara, MD,* Srividya Naganathan, MD,* Santhosh Eapen, MD*


*Jersey Shore University Medical Center, Neptune, NJ.

PRESENTATION
AUTHOR DISCLOSURE Drs Aikara and
Naganathan have disclosed no nancial
relationships relevant to this article. Dr Eapen
has disclosed that he owns common shares of
Dexcom, Roche Holdings, Omnipod, and
Illumina. This commentary does not contain a
discussion of an unapproved/investigative
use of a commercial product/device.

A 16-year-old Korean boy presents to the emergency department with acute onset
of lower-extremity weakness. He woke up from sleep, had several episodes of
emesis, and found he was unable to move his lower extremities. He recalls eating
half a dozen doughnuts earlier in the day. His parents found him struggling to get
up and brought him in for evaluation.
He reports intermittent palpitations and an unintentional weight loss of 40 lb
over the past year. He has had similar episodes of lower-extremity weakness with
no loss of sensation. These past episodes would last approximately 3 to 4 hours,
resolve spontaneously, and always occur at night. The patient was adopted at 6
months of age and, therefore, family history is unavailable.
On physical examination, his vital signs are temperature of 36.5C (97.8F),
heart rate of 78 beats per minute, respiratory rate of 16 breaths per minutes, and
blood pressure of 142/65 mm Hg. His body mass index is greater than the 95th
percentile. He is very combative and anxious. A grade 1 systolic ejection murmur
is auscultated at the left sternal border. He exhibits 5/5 muscle strength in the
upper extremities and 2/5 strength in the lower extremities. His sensation and
proprioception are intact, and his deep tendon reexes are difcult to elicit. The
rest of the physical examination ndings are within normal limits.
Initial laboratory evaluation documents potassium of 1.3 mEq/L (1.3 mmol/L)
and a urine drug screen positive for cannabinoids. His complete blood cell count,
creatinine phosphokinase, hepatic function panel, chest radiograph, and urinalysis results are within normal limits. Electrocardiography shows a normal sinus
rhythm with right bundle branch block and a prominent U wave consistent with
severe hypokalemia. Further diagnostic evaluation reveals the cause of the
hypokalemia and the explanation for the recurrent episodes of weakness.

DISCUSSION
The combination of muscle weakness and severe hypokalemia prompted an
evaluation for periodic paralysis. The typical signs and symptoms of thyrotoxicosis
occur as a result of the excess thyroid-stimulating hormone affecting the bodys
function and metabolism. Common symptoms include weight loss, nervousness
or irritability, diaphoresis, hyperthermia, heat intolerance, muscle weakness,
tachycardia, and tremors. This patient presented with a history of weight loss
and intermittent palpitations suggesting hyperthyroidism. Thyrotropin (TSH)

172

Pediatrics in Review
Downloaded from http://pedsinreview.aappublications.org/ by guest on April 2, 2016

measured during the paralytic event is low at 0.032 IU/mL


(normal range, 0.345.6 IU/mL). Further evaluation documents
elevated free triiodothyronine of 1,020 pg/dL (15.7 pmol/L)
(normal range, 227357 pg/dL [3.55.5 pmol/L]), free thyroxine
(T4) of 4.03 ng/dL (51.9 pmol/L) (normal range, 0.51.26 ng/dL
[6.416.2 pmol/l]), and a total T4 of 16.85 mg/mL (288 nmol/L)
(normal range, 5.289.87 mg/dL [90.3168.7 nmol/L]). He is
treated with potassium supplements and discharged from the
hospital with methimazole and propranolol. Follow-up evaluation shows elevated thyroid-stimulating immunoglobulin and
TSH receptor antibodies, supporting a diagnosis of Graves
disease. Since his admission, he has met with an endocrinologist. He is currently taking 5 mg of oral methimazole daily and
has had no further attacks; his repeat thyroid function test
results are within normal limits.

Differential Diagnoses
The inability to move muscles actively and voluntarily
against resistance is dened as weakness. Acute weakness
can be categorized based on location from central to peripheral (Table 1). Differential diagnoses for hypokalemia

TABLE 1.

include conditions and substances that cause a transcellular


shift or loss of potassium (Table 2).

The Condition
Periodic paralysis is a muscle disease due to a channelopathy.
Patients usually present with an episode of painless
muscle weakness. Common precipitants include heavy
exercise, fasting, and high-carbohydrate meals. Most cases
are hereditary, with an autosomal dominant inheritance
pattern. The acquired cases are usually associated with
hyperthyroidism.
Periodic paralysis is classied as hypokalemic and hyperkalemic, based on the serum potassium concentration.
Hypokalemic periodic paralysis occurs within the rst or
second decade, with attacks occurring infrequently but
lasting hours to days. Precipitants include exercise, carbohydrate load, and stress. Hyperkalemic periodic paralysis is seen
within the rst decade, with attacks occurring frequently and
lasting minutes to hours. Among the precipitants are exercise,
fasting, stress, and potassium-rich food. Both forms of paralysis
are associated with later-onset myopathy.

Differential Diagnoses for Weakness

CENTRAL

SPINAL CORD

PERIPHERAL NERVE

Intracranial hemorrhage

Cord trauma

Guillain-Barr syndrome

Subdural hematoma

Hematoma

Heavy metal toxins (arsenic,


mercury, thallium)

Epidural hematoma

Vertebral column fracture

Ciguatera sh poisoning

Rupture of an arteriovenous malformation or aneurysm

Spinal tumor

Paralytic shellsh poisoning

Subarachnoid hemorrhage

Paraspinal infection or inammation

Stroke

Epidural abscess

Brain tumor

Discitis

Seizure with Todd paralysis

Transverse myelitis

Hemiplegic migraine

Anterior horn cell disease


(ie, poliomyelitis)

Alternating hemiplegia of childhood


NEUROMUSCULAR JUNCTION

MUSCLE

OTHER

Botulism

Rhabdomyolysis

Electrolyte disturbances

Myasthenia gravis

Myositis

Medication-induced

Organophosphate poisoning

Pyomyositis

Conversion disorder

Carbamate poisoning

Familial periodic paralysis

Neurotoxic snake envenomation

Thyrotoxic periodic paralysis

Tick paralysis

Trichinellosis

Vol. 37 No. 4
Downloaded from http://pedsinreview.aappublications.org/ by guest on April 2, 2016

APRIL 2016

173

TABLE 2.

Differential Diagnoses for


Hypokalemia

RENAL LOSSES

TRANSCELLULAR SHIFTS

Interstitial nephritis

Thyrotoxic periodic paralysis

Barter syndrome

Familial periodic paralysis

Gitelman syndrome

Sporadic periodic paralysis

Renal tubular acidosis

Barium poisoning

Primary hyperaldosteronism

Alkalosis

Aminoglycosides

Tocolytics

Amphotericin

Theophylline toxicity

Chemotherapeutic agents

Chloroquine toxicity

Licorice

Insulin

Liddle syndrome

Diuretics

Treatment and Management

EXTRARENAL LOSSES

DECREASED INTAKE

Infectious diarrhea

Anorexia

Acute treatment may include potassium supplementation,


with careful monitoring for rebound hyperkalemia. Oral
potassium supplementation is the preferred mode of treatment for this reason. However, if serum potassium is less
than 2 mEq/L (2 mmol/L) and electrocardiographic changes
characteristic of hypokalemia are evident, intravenous
potassium is indicated. The suggested protocol is to administer 30 mEq of oral potassium every 2 hours until improvement begins, with a maximum of 90 mEq in 24 hours. Both
serum potassium and heart activity should be monitored.
Propranolol (1 mg intravenously or 3 mg/kg orally) is indicated for patients with symptoms of hyperthyroidism
and hypokalemia that persist despite potassium supplementation. Maintaining a euthyroid state and avoiding highglycemic meals can prevent repeated attacks.

Short bowel syndrome


Sweating
Laxative abuse

Thyrotoxic periodic paralysis occurs more commonly in


males, with a higher prevalence in Asians, usually of Chinese,
Japanese, Vietnamese, Filipino, or Korean descent. Although
attacks can occur at any time during the day, most tend to
occur at night or during the early morning hours, which has
prompted the terms nocturnal paralysis or night palsy.
Attacks occur in the presence of elevated thyroid hormone
and cease once values normalize. The exact mechanism of
thyrotoxic periodic paralysis is not well understood but is
believed to be related to thyroid hormone increasing tissue
responsiveness to b-adrenergic stimulation.
Most cases of hyperthyroidism are due to Graves disease,
but other potentially causative conditions are thyroiditis,
toxic nodular goiter, toxic adenoma, TSH-secreting pituitary
tumor, and exogenous ingestion of thyroid hormone or
iodine. The age of onset is typically 20 to 40 years but
has been described in adolescents. Clinical features of
hyperthyroidism may precede the onset of periodic paralysis
by months to years, occur at the same time, or develop after
the diagnosis is made. Although cases of thyrotoxic periodic
paralysis are characterized by tachycardia and hyperthermia, this patient did not exhibit these changes in vital
signs on presentation. His heart rate was 78 beats per minute,

174

which falls within the reference range. However, he may


have had a lower baseline resting heart rate. The patient
also presented 4 hours after initiation of the attack, and
compensatory mechanisms may have allowed normalization of his heart rate and temperature. Attacks are characterized as recurrent, transient episodes of muscle
weakness of the proximal muscles, primarily of the lower
extremities, that range from mild to complete accid
paralysis. The severity of the weakness usually corresponds to the degree of hypokalemia but not to the
thyrotoxic state.
The presence of hypokalemia in the setting of acute
weakness is a clue to evaluate for periodic paralysis. When
there is no family history, the possibility of thyrotoxicosis
must be evaluated.

Lessons for the Clinician


The vast array of differential diagnoses for acute paralysis
in children warrant a thorough evaluation.
Hypokalemia can result from a variety of causes and can
cause acute weakness. It is often a signal for the physician
to consider periodic paralysis.
The severity of paralysis correlates to the degree of
hypokalemia, irrespective of the thyroid hormone
values.
Thyrotoxicosis should be considered in the differential
diagnosis when adolescents present with recurrent episodes of weakness and hypokalemia.
Suggested readings for this article are at http://pedsinreview.
aappublications.org/content/37/4/172.

Pediatrics in Review
Downloaded from http://pedsinreview.aappublications.org/ by guest on April 2, 2016

Case 2: Acute Onset of Lower Extremity Weakness in a 16-year-old Korean Boy


Sandy Aikara, Srividya Naganathan and Santhosh Eapen
Pediatrics in Review 2016;37;172
DOI: 10.1542/pir.2015-0130

Updated Information &


Services

including high resolution figures, can be found at:


http://pedsinreview.aappublications.org/content/37/4/172

References

This article cites 14 articles, 4 of which you can access for free at:
http://pedsinreview.aappublications.org/content/37/4/172#BIBL

Permissions & Licensing

Information about reproducing this article in parts (figures, tables) or


in its entirety can be found online at:
http://beta.pedsinreview.aappublications.org/site/misc/Permissions.x
html

Reprints

Information about ordering reprints can be found online:


http://beta.pedsinreview.aappublications.org/site/misc/reprints.xhtml

Downloaded from http://pedsinreview.aappublications.org/ by guest on April 2, 2016

Case 2: Acute Onset of Lower Extremity Weakness in a 16-year-old Korean Boy


Sandy Aikara, Srividya Naganathan and Santhosh Eapen
Pediatrics in Review 2016;37;172
DOI: 10.1542/pir.2015-0130

The online version of this article, along with updated information and services, is
located on the World Wide Web at:
http://pedsinreview.aappublications.org/content/37/4/172

Pediatrics in Review is the official journal of the American Academy of Pediatrics. A monthly
publication, it has been published continuously since 1979. Pediatrics in Review is owned,
published, and trademarked by the American Academy of Pediatrics, 141 Northwest Point
Boulevard, Elk Grove Village, Illinois, 60007. Copyright 2016 by the American Academy of
Pediatrics. All rights reserved. Print ISSN: 0191-9601.

Downloaded from http://pedsinreview.aappublications.org/ by guest on April 2, 2016

Altered Mental Status in a 14-Year-Old Girl


Allison Lowe Guimera, MD,* Deepa Kulkarni, MD*
*UCLA Mattel Childrens Hospital, Los Angeles, CA.

PRESENTATION
AUTHOR DISCLOSURE Drs. Lowe Guimera
and Kulkarni have disclosed no nancial
relationships relevant to this article. This
commentary does contain a discussion of an
unapproved/investigative use of a
commercial product/device.

A 14-year old girl presents to the emergency department with a 1-day history of
altered mental status. She has severe autism spectrum disorder, learning
disability, and hyperactive behavior for which she takes risperidone and transdermal clonidine. She is ambulatory and nonverbal at baseline, but she has been
increasingly somnolent and unable to walk or eat over the past few hours. There
is no history of witnessed head trauma or seizure-like activity. She has no
associated fevers, respiratory symptoms, vomiting, or diarrhea.
On physical examination, her temperature is 36.6C (97.9F), heart rate is
53 beats per minute, respiratory rate is 10 breaths per minute, blood pressure is
88/52 mm Hg, and oxygen saturation is 100% in room air. She is somnolent,
minimally arousable, and opens her eyes briey to painful stimuli. Her head is
atraumatic, neck is supple, and pupils are pinpoint and minimally reactive to
light. She has regular cardiac rhythm, unlabored shallow breathing, and clear
lungs to auscultation. Her extremities are well perfused and atraumatic. Her
reexes are normal.
Laboratory evaluation reveals a normal complete blood cell count and
complete metabolic panel. Electrocardiography shows sinus bradycardia.
Computed tomography scan of the brain yields results within normal limits.
A urinary toxicology screen is negative. After administration of 2 normal
saline boluses and 2 doses of naloxone, she has minimal improvement of
hypotension and mental status. Additional evaluation and history reveal the
diagnosis.

DISCUSSION
After further questioning, her parents report that she frequently ingests nonfood
objects, and closer physical examination revealed that her transdermal clonidine
patch was absent, with evidence of excoriation at its previous site. She was
admitted to the hospital with presumed clonidine poisoning due to ingestion of
the patch.
Clonidine is an a-2 agonist that acts centrally in the brainstem to reduce
sympathetic outow, resulting in decreased peripheral vascular resistance, renal
vascular resistance, heart rate, and blood pressure. Clonidine has multiple clinical
indications for use in the pediatric population, including treatment of hypertension, attention-decit/hyperactivity disorder, conduct disorder, and Tourette
syndrome. It is frequently used off-label for other behavioral problems, sleep

Vol. 37 No. 4
Downloaded from http://pedsinreview.aappublications.org/ by guest on April 2, 2016

APRIL 2016

175

disturbance, headaches, postoperative nausea and vomiting,


opioid withdrawal, and pain modulation.
Clonidine is available in regular and extended-release
oral formulations as well as transdermal patches for more
sustained release. The transdermal patch contains a reservoir of medication that moves down a concentration gradient toward the patient. To maintain a sufcient gradient for
prolonged drug delivery, the reservoir contains an excess
amount of drug. In fact, more than 50% of the active drug is
present in patches removed after 7 days of wear, making
even disposed patches a threat for toxic ingestion.

The Condition
Clonidine overdose can occur from exploratory ingestion in
young children, suicidal ingestion, or administration error.
The severity of symptoms in an overdose is highly variable
and depends on the childs age, the formulation, the route of
exposure, and the dose delivered. Most signs of toxicity
develop 1 to 4 hours after oral formulation ingestion and
may continue past 4 hours for extended-release oral formulations or patch exposure. The symptomatic period following an ingested transdermal patch is unpredictable because
medication may be erratically released while passing through
the gastrointestinal tract.
Classically, clonidine overdose presents with symptoms
of early hypertension followed by hypotension, bradycardia,
respiratory depression, central nervous system depression,
and miosis. In larger overdoses, cardiac conduction defects,
apnea, coma, and seizures may occur, but death is rare.
These symptoms can easily be confused with other toxic
ingestions, including ethanol, opiates, benzodiazepines,
barbiturates, and sedative agents. Clonidine overdose also
has features similar to hypoglycemia, traumatic brain injury,
the postictal state, meningitis or encephalitis, acidemia, and
uremia. Given the broad differential diagnosis, a thorough
history and physical examination are essential before pursuing diagnostic studies.

Management
Initial management of all toxic ingestions begins with stabilizing the patient, focusing on airway, breathing, and circulation. Evaluation should include blood glucose, complete
metabolic panel, electrocardiography, urine toxicology screen,
and urine pregnancy test in adolescent females. All patients
also should be evaluated for coingestants such as aspirin and
acetaminophen. However, results vary, depending on time and
type of exposure, and may even be normal. Consultation with
poison control is, therefore, important if clinical suspicion is
high to assist with the recognition of a potential toxidrome.

176

Rapid identication is essential because several of the


initial treatments are time-sensitive. Specically, activated charcoal, if indicated, is most effective when given
within 1 hour of exposure to prevent systemic absorption.
In addition, dermal and ocular decontamination must be
performed expeditiously for toxins that have rapid absorption. Other methods of decontamination that should
be considered include whole bowel irrigation, which is
effective for nonadsorbed agents as well as sustainedrelease preparations. Certain agents may also benet
from urine alkalization or hemodialysis to enhance
elimination.
This patients altered mental status and depressed vital
signs raised the possibility of a toxidrome. Her blood
pressure was initially stabilized with normal saline boluses.
Atropine can be administered to reverse symptomatic bradycardia caused by atrioventricular node depression in
clonidine toxicity, but it was not needed in this case.
Naloxone was administered but had minimal effect, making opioid toxicity less likely. Clonidine toxicity has a
variable response to naloxone, mediated by clonidines
cross-reactivity with potassium channels on opioid m receptors. Dermal decontamination for this patient was ensured
with a thorough physical examination, which demonstrated
the absence of additional clonidine patches. Whole bowel
irrigation was required to remove the clonidine source and
prevent further gastrointestinal absorption. Elective intubation would have been required for safe administration via
the nasogastric route, given this girls depressed mental
status and risk of aspiration. Therefore, an enema was
performed instead, followed by a continuous docusate rectal
infusion. Two days after admission, the clonidine patch was
found in her stool. She recovered well and was discharged
the following day with normal vital signs and baseline
mental status.

Lessons for the Clinician


A thorough history and physical examination are essential
for all patients presenting with a suspected ingestion
because initial laboratory data may not lead to a denitive
diagnosis.
Transdermal medications, even after discarding, may be
applied or ingested by young children, leading to inadvertent overdoses.
Clonidine poisoning is rarely life-threatening and is
treated with decontamination, naloxone, and supportive
care.
Suggested readings for this article are at http://pedsinreview/
aappublications.org/content/37/4/175.

Pediatrics in Review
Downloaded from http://pedsinreview.aappublications.org/ by guest on April 2, 2016

Case 3: Altered Mental Status in a 14-Year-Old Girl


Allison Lowe Guimera and Deepa Kulkarni
Pediatrics in Review 2016;37;175
DOI: 10.1542/pir.2015-0164

Updated Information &


Services

including high resolution figures, can be found at:


http://pedsinreview.aappublications.org/content/37/4/175

References

This article cites 5 articles, 0 of which you can access for free at:
http://pedsinreview.aappublications.org/content/37/4/175#BIBL

Permissions & Licensing

Information about reproducing this article in parts (figures, tables) or


in its entirety can be found online at:
http://beta.pedsinreview.aappublications.org/site/misc/Permissions.x
html

Reprints

Information about ordering reprints can be found online:


http://beta.pedsinreview.aappublications.org/site/misc/reprints.xhtml

Downloaded from http://pedsinreview.aappublications.org/ by guest on April 2, 2016

Case 3: Altered Mental Status in a 14-Year-Old Girl


Allison Lowe Guimera and Deepa Kulkarni
Pediatrics in Review 2016;37;175
DOI: 10.1542/pir.2015-0164

The online version of this article, along with updated information and services, is
located on the World Wide Web at:
http://pedsinreview.aappublications.org/content/37/4/175

Pediatrics in Review is the official journal of the American Academy of Pediatrics. A monthly
publication, it has been published continuously since 1979. Pediatrics in Review is owned,
published, and trademarked by the American Academy of Pediatrics, 141 Northwest Point
Boulevard, Elk Grove Village, Illinois, 60007. Copyright 2016 by the American Academy of
Pediatrics. All rights reserved. Print ISSN: 0191-9601.

Downloaded from http://pedsinreview.aappublications.org/ by guest on April 2, 2016

in

Brief
Chagas Disease
Aaron W. Tustin, MD, MPH,* Natalie M. Bowman, MD, MPH
*Department of Environmental Health Sciences, Johns Hopkins Bloomberg School of Public Health, Baltimore, MD.

Department of Medicine, Division of Infectious Diseases, University of North Carolina at Chapel Hill, Chapel Hill, NC.

AUTHOR DISCLOSURE Dr Tustin has


disclosed no nancial relationships relevant to
this article. Dr Bowman has disclosed that she
receives grant support from NIH (NIAID) and
BWF/ASTMH to research Chagas disease
(Grants K23 AI113197-02, P30 AI50410, and
others). This commentary does contain a
discussion of an unapproved/investigative
use of a commercial product/device.

Congenital Chagas Disease:


Recommendations for Diagnosis,
Treatment and Control of Newborns,
Siblings and Pregnant Women. Carlier Y,
Torrico F, Sosa-Estani S, et al. PLoS Negl Trop
Dis. 2011;5(10):e1250
Trypanosoma cruzi and Chagas Disease in
the United States. Bern C, Kjos S, Yabsley MJ,
Montgomery SP. Clin Microbiol Rev.
2011;24(4):655-681
Congenital Transmission of Chagas
disease - Virginia, 2010. Centers for Disease
Control and Prevention (CDC). MMWR Morb
Mortal Wkly Rep. 2012;61(26):477-479
Congenital Chagas Disease: An Update.
Carlier Y, Sosa-Estani S, Luquetti AO, Buekens
P. Mem Inst Oswaldo Cruz. 2015;110
(3):363-368

Approximately 8 million people worldwide are infected by the protozoan parasite


Trypanosoma cruzi, the causative agent of Chagas disease. After a latent period that
can last years or decades, 10% to 30% of infected people develop serious complications, such as cardiomyopathy or gastrointestinal dysfunction. Contrary to popular
belief, Chagas disease is not solely a vector-borne infection of Latin America.
Clinicians in nonendemic regions must be aware of the potential for childhood
T cruzi infections.
Humans typically acquire the parasite through contact with the infected feces
of blood-feeding triatomine insects. Vector-mediated transmission occurs in
endemic regions that extend from the southern United States to the southern
cone of South America. T cruzi can also be transmitted in food and beverages
contaminated with triatomine feces, via infected organ transplants and blood
transfusions, and vertically from mother to child. In the United States, human
contact with triatomines is minimal, and blood donations are screened for T cruzi.
However, more than 300,000 US residents, mostly immigrants from Latin
America, may harbor chronic T cruzi infections. Many of those infected are
children and reproductive-age women.
Vertical transmission is an underrecognized problem. Children born to
infected mothers have an approximately 5% chance of acquiring the parasite.
Although there has been only one reported case of congenital T cruzi infection in
the United States (in an infant born in 2010 to a Bolivian mother), several
hundred undetected congenital T cruzi infections are estimated to occur in the
United States each year. Diagnosis is difcult because most infected newborns are
asymptomatic. A minority of infected infants demonstrate nonspecic signs and
symptoms, including low Apgar scores, low birthweight, respiratory distress,
hepatosplenomegaly, anasarca, pericardial and pleural effusions, and meningoencephalitis. The presentation can easily be confused with neonatal sepsis or
TORCH infections (which include toxoplasmosis, rubella, cytomegalovirus, and
herpes viruses). Clinicians may overlook T cruzi in the differential diagnosis,
especially in nonendemic areas.
To improve detection of congenital infections, the World Health Organization
recommends antenatal T cruzi screening in all pregnant women who have ever
lived or received a blood transfusion in an endemic region. Infected women are
not treated prenatally to prevent vertical transmission because antitrypanosomal
medications are contraindicated during pregnancy and breastfeeding. Instead,
children born to T cruzi-infected mothers should be tested during infancy and
treated if necessary. Unfortunately, prenatal screening recommendations are not
widely followed, even in endemic countries. Many at-risk infants are born to
mothers with unknown T cruzi status. Pediatricians should consider testing these
newborns for Tcruzi, especially if they are symptomatic. Diagnostic testing should

Vol. 37 No. 4
Downloaded from http://pedsinreview.aappublications.org/ by guest on April 2, 2016

APRIL 2016

177

be offered to siblings of any child diagnosed with Chagas


disease, and the mother should also be referred for treatment evaluation.
Newborns at risk for T cruzi infection should be tested
with polymerase chain reaction (PCR), the preferred diagnostic technique in children younger than age 9 months. PCR is
more sensitive than microscopy-based techniques to detect
T cruzi in blood, although it may not be available in resourcelimited areas. If the initial test result is negative, PCR should
be repeated at 4 to 6 weeks of age because parasitemia may
increase over the rst postnatal month. Infants who lack
detectable parasitemia should undergo serologic testing after
age 9 months to conrm the absence of infection. Serologic
methods such as enzyme-linked immunosorbent assay
(ELISA) and immunouorescent antibody test (IFA) should
not be used in children younger than 9 months, in whom
circulating maternal antibodies can cause false-positive
results. After 9 months of age, serology becomes the diagnostic method of choice because parasitic DNA is often
undetectable by PCR during the chronic stage of infection.
Positive serology should be conrmed with a second assay
because these tests have imperfect sensitivity and specicity.
All children diagnosed with T cruzi infection should
receive antitrypanosomal therapy with either benznidazole
or nifurtimox. Although these medications are not approved
by the US Food and Drug Administration (FDA) in the
United States, they can be obtained from the Centers for
Disease Control and Prevention under investigational permits. Standard therapy is a 60- to 90-day course of benznidazole 5 to 7 mg/kg per day or nifurtimox 10 to 15 mg/kg per
day divided in 2 to 3 daily doses. Both drugs have a high rate
of adverse effects but are tolerated better in children than

178

adults. Cure rates of nearly 100% are observed in congenitally


infected infants who receive treatment within the rst postnatal year. The cure rate decreases to approximately 60% in
older children with chronic infections. Prompt detection and
treatment of congenital T cruzi infection is, therefore, essential to avoid future complications.
COMMENT: I found this In Brief fascinating because I
have not been familiar with the presentation of Chagas
disease nor considered it in the differential diagnosis of
patients. Yet, the World Health Organization estimates that
Chagas disease is the most prevalent parasitic disease in the
American continents, including North, Central, and South
America.
As the authors noted, the presentation in newborns is
diverse, making diagnosis challenging for those who are not
familiar with the wide range of possibilities. Congenital
Chagas disease must be considered in infants born to mothers who are from endemic areas, such as the southern portion
of South America: Brazil, Paraguay, Argentina, and Bolivia.
Early identication and treatment can prevent progression to
more serious manifestations associated with chronic infection. Chronic Chagas disease can affect the heart, leading to
cardiomyopathy, associated arrhythmias, and cardiac dysfunction. Although less common, gastrointestinal involvement can affect the esophagus or colon and lead to motility
disorders. Because treatment is available and effective,
diagnosis in both congenital and chronic states is important to patient quality of life and prevention of resultant
morbidity.

Pediatrics in Review
Downloaded from http://pedsinreview.aappublications.org/ by guest on April 2, 2016

Janet Serwint, MD
Associate Editor, In Brief

Chagas Disease
Aaron W. Tustin and Natalie M. Bowman
Pediatrics in Review 2016;37;177
DOI: 10.1542/pir.2015-0116

Updated Information &


Services

including high resolution figures, can be found at:


http://pedsinreview.aappublications.org/content/37/4/177

References

This article cites 4 articles, 1 of which you can access for free at:
http://pedsinreview.aappublications.org/content/37/4/177#BIBL

Permissions & Licensing

Information about reproducing this article in parts (figures, tables) or


in its entirety can be found online at:
http://beta.pedsinreview.aappublications.org/site/misc/Permissions.x
html

Reprints

Information about ordering reprints can be found online:


http://beta.pedsinreview.aappublications.org/site/misc/reprints.xhtml

Downloaded from http://pedsinreview.aappublications.org/ by guest on April 2, 2016

Chagas Disease
Aaron W. Tustin and Natalie M. Bowman
Pediatrics in Review 2016;37;177
DOI: 10.1542/pir.2015-0116

The online version of this article, along with updated information and services, is
located on the World Wide Web at:
http://pedsinreview.aappublications.org/content/37/4/177

Pediatrics in Review is the official journal of the American Academy of Pediatrics. A monthly
publication, it has been published continuously since 1979. Pediatrics in Review is owned,
published, and trademarked by the American Academy of Pediatrics, 141 Northwest Point
Boulevard, Elk Grove Village, Illinois, 60007. Copyright 2016 by the American Academy of
Pediatrics. All rights reserved. Print ISSN: 0191-9601.

Downloaded from http://pedsinreview.aappublications.org/ by guest on April 2, 2016

in

Brief
Dengue and Chikungunya
Paul J. Lee, MD,* Leonard R. Krilov, MD
*Childrens Medical Center, Winthrop-University Hospital, Mineola, NY.

State University of New York at Stony Brook School of Medicine, Stony Brook, NY.

AUTHOR DISCLOSURE Dr Lee has disclosed


that he was a speaker for Novartis Vaccines.
His relationship with them ended in June
2015. Dr Krilov has disclosed that he is site
principal investigator for a meningococcal B
vaccine trial for Pzer and that Pzer provides
funding to his institution; site principal
investigator for an observational study of
respiratory syncytial virus (RSV) infections in
preterm infants for AstraZeneca and that
AstraZeneca provides funding to his
institution; and site principal investigator for a
clinical trial of a humanized monoclonal
antibody to prevent RSV in high-risk preterm
infants for Regeneron. This commentary does
not contain discussion of an unapproved/
investigative use of a commercial product or
device.

CDC Health Information for International


Travel 2016. Centers for Disease Control and
Prevention. New York, NY: Oxford University
Press; 2016. Also available at: http://wwwnc.
cdc.gov/travel/page/yellowbook-home.
Accessed October 31, 2015
Dengue. Guzman MG, Harris E Lancet.
2015;385(9966):453465 doi: 10.1016/S01406736(1460572-9
Dengue. Simmons CP, Farrar JJ, Nguyen V,
Wills B. N Engl J Med. 2012;366(15):14231432.
doi: 10.1056/NEJMra1110265
Infectious Disease Emergencies in Returning
Travelers: Special Reference to Malaria,
Dengue Fever, and Chikungunya. Wattal C,
Goel N. Med Clin North Am. 2012;96
(6):12251255. doi: 10.1016/j.mcna.2012.08.004
Chikungunya Virus and the Global Spread
of a Mosquito-borne Disease. Weaver SC,
Lecuit M. N Engl J Med. 2015;372(13):
12311239. doi: 10.1056/NEJMra1406035

Dengue is caused by 4 related but distinct aviviruses, DENV1 4, which result


in 390 million infections annually, with one-third of the worlds population at
risk. Most aviviruses are transmitted by arthropods, such as mosquitoes and
ticks, and are responsible for diseases such as yellow fever, West Nile fever, and
Japanese encephalitis. DENV is primarily transmitted by the bite of an infected
female Aedes aegypti mosquito, which is endemic to every global region except
Europe and Antarctica. Dengue primarily occurs in South and Southeast Asia,
Africa, tropical South and Central America, and the Caribbean. In the United
States, dengue is a nationally notiable disease, with 677 cases reported to the
Centers for Disease Control and Prevention (CDC) in 2014. It is endemic in
Puerto Rico, the US Virgin Islands, and American Samoa. Most US cases result
from travel to an endemic area, but locally acquired dengue has been documented
in Texas and south Florida.
Seventy-ve percent of DENV infections in all age groups are asymptomatic,
but a spectrum of disease is seen in the remaining 96 million patients, ranging
from self-limited fever to life-threatening shock. Classic dengue fever begins 4 to 7
days after an infected mosquitos bite and typically presents as a ulike illness
with a temperature greater than 38.5C (101.3F), headache, retro-orbital pain,
and severe myalgias, from which its eponym, break-bone fever, arose. A ne,
transient macular rash, nausea, and vomiting may also occur. However, infection
may present atypically; the classic ndings occur in fewer than 60% of patients.
With the exception of high temperatures, children are often less symptomatic
than adults during the rst phase of infection, which lasts 3 to 7 days. Physical
examination may reveal a palpable liver and petechiae and bruising, caused by
moderate thrombocytopenia. Other laboratory ndings include leukopenia and
an aspartate aminotransferase value less than 1,000 U/L (16.7 mkat/L).
Most patients subsequently recover without complications, but about 1%,
usually children and young adults, develop systemic vascular leak after defervescing. This leakage causes increased hemoconcentration, hypoproteinemia, pleural
effusions, and ascites, resulting in more serious disease, referred to as dengue
hemorrhagic fever (DHF) or dengue shock syndrome (DSS). DHF is dened by
a triad of ndings: plasma leakage, thrombocytopenia, and hemorrhage. The
bleeding is most notable in the skin, but hematemesis, melena, menorrhagia, and
epistaxis can also occur. Children rarely have clinically signicant bleeding,
but adults can have substantial hemorrhage, even with minor plasma leakage.
DSS occurs in patients with DHF when the pulse pressure narrows to less than
20 mm Hg from plasma volume loss and peripheral vascular collapse begins. DSS
is particularly insidious because patients initially appear well, with normal or
elevated systemic blood pressure. However, once they develop hypotension,
irreversible shock and death can occur despite aggressive resuscitation. Clinicians

Vol. 37 No. 4
Downloaded from http://pedsinreview.aappublications.org/ by guest on April 2, 2016

APRIL 2016

179

should closely monitor patients for signs of impending


shock, such as persistent vomiting, increasingly severe
abdominal pain, tender hepatomegaly, mucosal bleeding,
lethargy, or restlessness. Helpful diagnostic indicators are
the presence of effusions and increasing or high hematocrit
as the platelet count rapidly declines. The altered vascular
permeability quickly resolves after 48 to 72 hours, and its
pathogenesis remains unclear. DHF and DSS do not occur
with the initial DENV infection.
Immunity to one serotype after infection may predispose
individuals infected with another serotype to severe dengue.
Therefore, most travelers, except for those frequently visiting family and relatives, are unlikely to develop what the
World Health Organization now classies as severe dengue.
Dengue is usually diagnosed clinically because most
endemic areas lack access to clinical laboratories. Even in
developed countries, conrmation is slow and limited to
specialized reference laboratories. The gold standard for
diagnosis, although it is not helpful in patient care, remains
testing paired acute and convalescent (obtained 10 to 14 days
later) serum with a DENV hemagglutinin inhibition assay.
Newer tests, such as DENV nonstructural protein 1 (NS1)
or immunoglobulin (Ig)M enzyme-linked immunosorbent
assays or RNA reverse transcription-polymerase chain reaction (RT-PCR), can now diagnose dengue quickly.
Dengue management is largely supportive. Early recognition of shock and careful, frequent assessment of pulse,
blood pressure, hematocrit, and urine output, with administration of oral and intravenous uids and blood products to
maintain adequate intravascular volume, is critical. Currently there is no effective antiviral agent for DENV. Prevention is based on mosquito control measures, although a
recombinant live attenuated tetravalent vaccine (CYD-TDV)
may be approved in 2016.
Chikungunya fever is caused by an alphavirus (CHIKV),
which was isolated during an epidemic in 1952 in Tanzania.
Its name is the Makonde word for that which bends upward, describing the bent posture patients that assume
from the severe arthralgia CHIKV causes. In 2004, CHIKV
made headlines when it spread from Africa to South Asia,
causing a number of epidemics affecting millions of people.
In October 2013, CHIKV appeared in St. Martin and rapidly
spread throughout the Caribbean and Central America, with
1.7 million cases reported within 2 years, 3,270 of which
were reported in the United States.
Although CHIKV is spread by the same mosquitoes as
DENV, it can be more readily transmitted. Paired with the

180

vulnerability of a population that has no prior immunity


to CHIKV, efcient transmissibility has led to large-scale
outbreaks. One outbreak in northeastern Italy in 2007
was caused by the local Aedes aegypti and Aedes albopictus
mosquitoes biting an infected traveler and then infecting
the local population with an adapted CHIKV that had
enhanced transmissibility. Both of these mosquito species are present in the United States, and at least 11
endemic cases have been documented. Estimates of CHIKV
incidence and prevalence are not as well dened as for
DENV.
In sharp contrast to DENV, 85% of CHIKV infections are
symptomatic, with rapid onset of a temperature of 39 to 40C
(102.4 to 104F) after a brief incubation of 2 to 4 days.
Polyarthralgia begins 2 to 5 days after fever, frequently
involving more than 10 joint groups. The arthralgia tends
to be symmetric and more common in distal than proximal
joints, especially large joints, although the axial skeleton
may be involved. Patients are often immobilized by pain.
Macular or maculopapular rash is also common, appearing
3 or more days after fever and lasting 3 to 7 days. The rash
begins on the trunk or extremities, can be patchy or diffuse,
may involve the face as well as the palms and soles, and may
be pruritic in up to 50% of patients. Children may develop
bullous lesions, but petechiae and bruising are uncommon.
Gastrointestinal symptoms and erythematous ears from
CHIKV chondritis occur as well.
Similar to DENV, CHIKV is usually diagnosed clinically.
Marked lymphopenia is characteristic as well as thrombocytopenia, elevated aspartate aminotransferase/alanine
aminotransferase, and hypocalcemia. RT-PCR can diagnose
CHIKV in the rst week of illness, and IgM can be positive
before the fth day of illness and persist for months. A
fourfold rise in IgG between acute and convalescent serum
also conrms CHIKV. Treatment of CHIKV is supportive,
and prevention is through mosquito control, with vaccine
research still in early stages.
Differentiating DENV and CHIKV can be difcult because both are acute systemic febrile illnesses with rash that
occur in the same world regions. The Table highlights important differences between the two. The differential diagnosis also includes malaria, rickettsial disease, typhoid,
leptospirosis, Epstein-Barr virus infection, measles, rubella,
and other hemorrhagic fever viruses.
COMMENT: Dr Anthony Fauci, Director of the National
Institute of Allergy & Infectious Diseases, has called mosquitoes the most destructive animals on earth, without any

Pediatrics in Review
Downloaded from http://pedsinreview.aappublications.org/ by guest on April 2, 2016

TABLE.

Differentiating Dengue from Chikungunya


DENGUE

CHIKUNGUNYA

Fever

Longer duration (57 days)

High-grade, shorter duration (35 days)

Arthralgia

Common

Common, but characteristically severe polyarthralgia

Rash

Fine, faint transient macules; spares the


palms/soles

Maculopapular, can be bullous, pruritic, and on the


face, palms/soles

Petechiae/ecchymoses

Common

Uncommon

Joint swelling

Very uncommon

Common

Abdominal pain

Common

Uncommon

Respiratory symptoms

Can occur

Not seen

Leukopenia

Common

Uncommon

Symptoms

Resolve within 1 week, except for fatigue

Frequently persist

Severe complications

Can occur

Rare

apparent redeeming characteristics. Within the family of


mosquitoes, the Aedes, along with the Anopheles (the vector
for malaria), is at the top of the list of villains. In addition to
dengue and chikungunya, Aedes species carry the arboviruses responsible for yellow fever, West Nile fever, eastern
equine encephalitis, and now the most recent addition to the
catalog of pandemic threats: Zika virus.
Initially identied serendipitously in Uganda in 1947, the
Zika virus spread locally in West Africa and then eastward
to the Pacic, causing outbreaks of infection in Micronesia
and Southeast Asia that were characterized by fever, rash,
arthralgias, and conjunctivitis. Until recently it raised no
special concern because the virus appeared to cause only
transient illness, if any at all; up to 80% of infected individuals remained asymptomatic. However, in 2015, if I may
paraphrase W.B. Yeats, all changed, changed utterly; a terrible pandemic was born.
By the years end, Brazil reported more than 1 million
cases of Zika infection and with them came an alarming
rise in cases of microcephaly as well as of Guillain-Barr
syndrome. Brazil hosted the World Cup in 2014, and some
have speculated that one of the myriad visitors from around
the globe transported the Zika virus as unwanted baggage.
When an infected human is bitten by an Aedes mosquito, the
insect ingests the virus, and its next bite passes the virus on
to another human host and so an epidemic is born. When

newly infected travelers return to homelands infested with


Aedes, the epidemic becomes a pandemic. More than 20
countries in the Americas have now reported indigenous
cases of Zika virus. As of this writing, no locally acquired
Zika infections have been reported in the United States, but
several travelers returning from areas of active infection
have been identied with the virus, and possibly some
southern states harbor mosquitoes that could become a
reservoir for the regional spread of Zika.
The Centers for Disease Control and Prevention have
urged women who are pregnant or trying to become pregnant to avoid travel to areas where Zika virus has been
identied and has issued guidelines for the management
of pregnant women who may have been infected and for
the evaluation of infants with possible congenital infection.
Early in 2016, the World Health Organization (perhaps
belatedly) declared the neurologic complications associated
with Zika virus infection to be an international health
emergency, which is the same classication it gave to the
Ebola epidemic in West Africa several years ago. As pediatricians, we know the devastating consequences of microcephaly. It is a terrible price to pay, even for the miracle of
modern air travel.
Henry M. Adam, MD
Associate Editor, In Brief

Vol. 37 No. 4
Downloaded from http://pedsinreview.aappublications.org/ by guest on April 2, 2016

APRIL 2016

181

Dengue and Chikungunya


Paul J. Lee and Leonard R. Krilov
Pediatrics in Review 2016;37;179
DOI: 10.1542/pir.2015-0172

Updated Information &


Services

including high resolution figures, can be found at:


http://pedsinreview.aappublications.org/content/37/4/179

References

This article cites 3 articles, 0 of which you can access for free at:
http://pedsinreview.aappublications.org/content/37/4/179#BIBL

Permissions & Licensing

Information about reproducing this article in parts (figures, tables) or


in its entirety can be found online at:
http://beta.pedsinreview.aappublications.org/site/misc/Permissions.x
html

Reprints

Information about ordering reprints can be found online:


http://beta.pedsinreview.aappublications.org/site/misc/reprints.xhtml

Downloaded from http://pedsinreview.aappublications.org/ by guest on April 2, 2016

Dengue and Chikungunya


Paul J. Lee and Leonard R. Krilov
Pediatrics in Review 2016;37;179
DOI: 10.1542/pir.2015-0172

The online version of this article, along with updated information and services, is
located on the World Wide Web at:
http://pedsinreview.aappublications.org/content/37/4/179

Pediatrics in Review is the official journal of the American Academy of Pediatrics. A monthly
publication, it has been published continuously since 1979. Pediatrics in Review is owned,
published, and trademarked by the American Academy of Pediatrics, 141 Northwest Point
Boulevard, Elk Grove Village, Illinois, 60007. Copyright 2016 by the American Academy of
Pediatrics. All rights reserved. Print ISSN: 0191-9601.

Downloaded from http://pedsinreview.aappublications.org/ by guest on April 2, 2016

Syncope
Bryan Cannon, MD,* Philip Wackel, MD*
*Mayo Clinic, Rochester, MN

Educational Gap
Syncope is a common problem in children and adolescents, but the
diagnostic yield for most tests used in its evaluation in pediatric patients is
low, and testing should be guided by a careful history and physical
examination. (1)

Objectives

After completing this article, readers should be able to:

1. List the most common causes of syncope in pediatric patients.


2. Understand the elements in the history or physical examination that
may be related to a life-threatening cause.
3. Describe the basic evaluation for a patient who presents with syncope.
4. Discuss basic education principles for patients who present with
uncomplicated syncope.
5. Screen and identify patients who require referral to a pediatric
subspecialist.

CASE PRESENTATIONS
Case 1
A 14-year-old boy with no signicant past medical history presents to the clinic
following a syncopal episode. He reports that he had been standing in church and
felt lightheaded before passing out. The event was witnessed and his parents
describe brief seizure-like activity when he was syncopal. He woke up 2 seconds
after he passed out and was alert and oriented. What is the most likely cause of the
syncope? What further evaluation is necessary? What recommendations would
you make for this patient? Would you refer him to a pediatric cardiologist or
neurologist?

AUTHOR DISCLOSURE Dr Cannon has


disclosed that he is a consultant for Medtronic,
USA, and serves on the Board of Directors for
Mayo Support Services-Texas. Dr Wackel has
disclosed no nancial relationships relevant to
this article. This commentary does not contain
a discussion of an unapproved/investigative
use of a commercial product/device.

Case 2
A 14-year-old girl with no signicant past medical history presents to the clinic
following a syncopal episode. She explains that she was running a 100-meter dash
and passed out in the middle of running. She had no symptoms before her
syncope. She woke up after 4 to 5 seconds but was very confused and did not
recognize her track coach or teammates. What are the potential causes of the
syncope? What further evaluation is necessary? What recommendations would

Vol. 37 No. 4
Downloaded from http://pedsinreview.aappublications.org/ by guest on April 2, 2016

APRIL 2016

159

you make for this patient? Would you refer her to a pediatric
cardiologist or neurologist?

DEFINITION
Syncope is a sudden and transient loss of consciousness and
postural muscle tone that reverses without intervention.
Normal brain function depends on a constant supply of
oxygen and glucose. A temporary decrease in cerebral blood
ow or glucose supply may result in transient loss or near
loss of consciousness.

EPIDEMIOLOGY
Syncope occurs in up to 50% of the general population, and
approximately 15% to 25% of children and adolescents
experience at least 1 episode of syncope before adulthood.
The incidence peaks in the late teenage years and occurs
more commonly in females. Syncope accounts for as many
as 3% of all emergency department visits in the pediatric
population. Approximately 60% of girls and 50% of boys
have more than 1 episode of syncope. (2) Syncope may result
from circulatory, metabolic, psychiatric, or neurophysiologic processes, and the causes range from benign to lifethreatening. Although a substantial portion of syncope in
the adult population is due to cardiac causes, most syncope
in the pediatric population is benign.

PATHOGENESIS
Syncope can be divided into several broad categories (Fig 1).
The most common form of syncope is autonomic-mediated

reex syncope, which accounts for approximately 75% of all


cases of pediatric syncope. (3) Reex syncope can be divided
into 3 basic types: vasovagal syncope, postural orthostatic
tachycardia syndrome (POTS), and orthostatic hypotension.

Vasovagal Syncope
Vasovagal syncope is also referred to as neurocardiogenic
syncope, vasodepressor syncope, or simple/common faint.
Typically, a prodrome lasts a few seconds to 1 minute and is
followed by syncope that usually lasts less than 1 minute.
Upright posture results in venous pooling in the lower
extremities (up to 25% of the total blood volume). Decreased
venous return results in lower blood pressure and stroke
volume (up to 40%). Specialized receptors, known as
C-bers, in the ventricle and atrium as well as mechanoreceptors in the carotid sinus and pulmonary arteries sense
stretch and pressure. C-ber/mechanoreceptor activation
results in a reexive increase in parasympathetic tone
(decreased heart rate and blood pressure), thereby reducing
pressure and stretch on the heart. Similarly, less C-ber/
mechanoreceptor activation from a decrease in blood
pressure results in a reexive increase in sympathetic
tone (increased heart rate and blood pressure). However,
in susceptible individuals, reduced venous return (ie,
decreased preload) results in a large increase in the force
of ventricular contractions to maintain adequate cardiac
output. The combination of increased contractility and a
decrease in ventricular blood volume creates the so-called
empty heart syndrome. An increase in endogenous catecholamines may further accentuate this response. Forceful
contractions by an underlled heart result in inappropriate
activation of the C-bers in the heart. The C-bers increase
Figure 1. Causes and categorization of
syncope.

160

Pediatrics in Review
Downloaded from http://pedsinreview.aappublications.org/ by guest on April 2, 2016

parasympathetic activity and inhibit sympathetic activity


(Bezold-Jarisch reex), which results in bradycardia, vasodilation, and hypotension. This cascade of events may occur
suddenly, with the consequent reduction of cerebral perfusion producing loss of consciousness.
Often, the patient assuming an upright posture initiates
the previously described cascade of events. A normal physiologic response initially may occur after assuming an
upright posture, but susceptible individuals may develop
an irregular sinus rate (sinus arrhythmia). They then can
abruptly develop symptomatic hypotension and bradycardia, with the systolic blood pressure decreasing up to 40 to
80 mm Hg. Bradycardia may progress to asystole that lasts 3
to 40 seconds in 4% to 6% of patients (Fig 2). Frequently,
sinus tachycardia is seen before the pause and an irregular

rhythm (sinus arrhythmia) is present after a spontaneous


rhythm returns. Typical symptoms of presyncope are dizziness, vision changes, a warm feeling, nausea, or an
irregular heartbeat before complete loss of consciousness,
although on rare occasions there are no presyncopal warning signs. Patients frequently have isolated episodes of
presyncope without subsequent loss of consciousness. During the syncopal event, after losing postural tone and
assuming a supine position, venous return from the lower
extremities increases. The increased cardiac blood volume
combined with a spontaneous increase in heart rate results
in increased cardiac output and a return to full consciousness within a brief period of time (typically less than 1 minute,
although unconsciousness may last longer than 5 minutes).
(3) Repeated episodes of syncope may occur if the patient

Figure 2. Asystolic pause characteristic of


reex syncope.

Vol. 37 No. 4
Downloaded from http://pedsinreview.aappublications.org/ by guest on April 2, 2016

APRIL 2016

161

attempts to sit up or stand too quickly or is held in an upright


position. Because vasovagal syncope is related to blood pooling in the lower extremities due to gravity, this type of syncope
almost never happens in the supine position, although it may
occur with sitting or standing or more commonly with a
change in position from supine to sitting, supine to standing,
or sitting to standing.

Postural Orthostatic Tachycardia Syndrome


POTS is a condition in which the heart rate increases
more than 30 beats per minute, with a heart rate of more
than 120 beats per minute without hypotension (inappropriate sinus tachycardia) within 10 minutes of assuming an upright position. Symptoms can be syncopal or
presyncopal.
POTS has been associated with chronic fatigue, gastrointestinal problems, headaches, poor sleep, difculty
concentrating, and decreased exercise tolerance. In some
patients, there is a substantial psychological inuence
to the symptoms. POTS has a female-to-male ratio of
5:1. Many affected patients do not have true syncope
but may have marked debilitation from presyncopal
symptoms.

Orthostatic Hypertension
Orthostatic hypotension represents a decrease in systolic
blood pressure of more than 20 mm Hg or in diastolic
pressure of more than 10 mm Hg within 3 minutes of
assuming an upright position. (4) In this condition, the
normal adrenergic vasoconstriction of arterioles and veins is
absent or inadequate, which results in hypotension without
a reex increase in heart rate. This can occur with prolonged
bed rest or prolonged standing or in patients who have
chronic medical conditions. Pure orthostatic hypotension
is a rare cause of syncope; frequently some element of
heart rate change accompanies the symptoms. Causes of
orthostatic hypotension include volume depletion, acute or
chronic illness, medications (eg, calcium channel blockers,
angiotensin-converting enzyme inhibitors), and autonomic
dysfunction.

Situational Syncope
A second form of syncope that is closely related to reex
syncope is situational syncope. The pathophysiology is
similar to vasovagal syncope, but a specic trigger or action
initiates the cascade of events leading to syncope. Common
forms of this syncope include syncope induced by the sight
of blood, pain, or fear. Hair brushing and micturition have
also been well described as triggers for syncope. The
episodes have a clinical presentation that is the same as

162

vasovagal syncope, with a prodrome and brief period of


unconsciousness.

Cardiac Syncope
The most concerning type of syncope is true cardiac syncope. Cardiac syncope is characterized by inadequate cerebral perfusion due to failure of the heart as a pump. Longer
episodes of cardiac pump failure can result in sudden death
or neurologic damage from prolonged ischemia, making
recognition of this type of syncope very important. True
cardiac syncope can be divided into 3 basic categories:
structural heart disease, arrhythmias (both tachyarrhythmias and bradyarrhythmias), and myocardial dysfunction.
The causes of cardiac syncope are detailed in Figure 3.
Cardiac syncope frequently occurs without any warning
signs, and any episode of loss of consciousness without
symptoms of presyncope should be assumed to have a
cardiac cause until proven otherwise. Cardiac syncope can
also be preceded by severe chest pain or very rapid heart
rates. Any patient who has no pulse during syncope or
requires cardiopulmonary resuscitation (CPR) or debrillation should undergo a full evaluation to exclude a cardiac
cause. One of the most concerning signs for underlying
cardiac pathology is syncope while actively exercising.
During exercise, the contracting leg muscles act as a pump
that helps return venous blood to the heart. Because of the
augmented venous return, vasovagal syncope during active
exercise can occur, although this is rare and cardiac syncope is
much more likely. In contrast, syncope immediately after
exercise can have a cardiac cause, but it is more likely to be
vasovagal because the increased blood ow to the muscles
during exercise combined with the abrupt withdrawal of the
increased venous return caused by muscular contractions may
set up the cascade of events leading to a vasovagal episode.

Neurologic Syncope
Neurologic conditions such as seizures or migraines can
also present with syncope. Patients who exhibit tonic-clonic
movements or have prolonged periods of confusion or
postictal states should undergo neurologic evaluation. However, brief posturing or myoclonic jerks that may be interpreted as seizures are common after an episode of reex
syncope. Such myoclonic jerks are typically of a short
duration; longer episodes are more likely related to seizure
activity. Loss of bowel or bladder control is rare in reex
syncope but more common in neurologic syncope. Cerebrovascular occlusive disease (stroke) is extremely rare in
children but may be seen in hypercoagulable states or
conditions that affect the arteries supplying blood to the
brain.

Pediatrics in Review
Downloaded from http://pedsinreview.aappublications.org/ by guest on April 2, 2016

Figure 3. Causes of cardiac syncope.

Psychogenic Syncope
Psychogenic syncope, also called pseudoseizures or
behavioral spells, occurs when no physiologic alterations lead to the syncopal episode. Psychogenic syncope
typically occurs in the presence of an audience or at a
specic time (before school). These episodes frequently
occur in an emotionally charged setting or during times
of stress. Typically, the patient has no pallor or hypotension during the episode, and the episodes may be prolonged, lasting up to 1 hour. Psychogenic syncope is very
rare before age 10 years. Usually the episodes are due to a
conversion disorder, in which adolescents express emotional feelings through physical symptoms, an expression
that is not a conscious or deliberate act. However, sometimes adolescents deliberately feign syncope to gain
attention or avoid a particular circumstance. Both emotional and sexual abuse may be the precipitating factor for
psychogenic syncope, and questions related to the potential for abuse should be raised in all patients with psychogenic syncope.

Other Causes of Syncope


Other causes of syncope include alcohol intoxication or drug
effects. Eating disorders such as anorexia may also result in
syncope related to a combination of bradycardia, hypovolemia, and hypoglycemia. Breathholding spells can cause
syncope in young patients. The patient who is angry, in
pain, or upset holds his or her breath until passing out. The

patient can appear cyanotic or pallid during the episode, but


the actual breathholding may be difcult to see. Typically
within seconds of syncope, a patient with a breathholding
spell begins breathing and regains consciousness. Breathholding spells usually occur in young children beginning as
young as 6 to 18 months of age but generally resolve by 5 to 6
years of age. Failure of venous return as a mechanism of
syncope can result from increased intrathoracic pressure
(pneumothorax, pericardial effusion) or severe hypovolemia, but these are very rare causes of syncope in the
pediatric population. Finally, teenagers may deliberately
engage in games designed to cause syncope, such as the
fainting lark, in which the teenager squats, hyperventilates, stands up suddenly, and forcefully exhales against a
closed glottis. Persistent hyperventilation can also result in
syncope.

CLINICAL ASPECTS
In the pediatric population, the overwhelming majority of
syncopal events are benign. However, a small subset of
patients is potentially at risk for sudden cardiac death,
making the evaluation of a patient with syncope a potential
diagnostic challenge. Clinicians should try to determine
an underlying cause to address the mechanism-specic
method of treatment and determine the potential risk for
a life-threatening event. A suggested algorithm is shown in
Figure 4.

Vol. 37 No. 4
Downloaded from http://pedsinreview.aappublications.org/ by guest on April 2, 2016

APRIL 2016

163

Figure 4. Diagnosis and treatment algorithm for true syncope.

History
The most important tool in diagnosing syncope is the
history. A detailed history of all the events surrounding the
episode is frequently time-consuming but critical in determining the potential underlying cause. It is important for
the patient to describe everything that he or she felt before
and after the episode. Talking to any witnesses can help to
determine the specics about the syncope, including the
duration. Of note, though, observers frequently overestimate the amount of time that a patient is unconscious.
Other important factors include time of day the syncope
happened and whether the patient had anything to eat or
drink that day because reex syncope may be brought on
by dehydration or fasting. Episodes of self-resolving syncope lasting longer than 10 minutes are almost never

164

physiologic and should raise concern for a psychological


mechanism.
An equally important part of the history is the family
history. Many life-threatening arrhythmias are caused by an
autosomal dominant genetic condition. A history of sudden
cardiac death or death in a relative younger than age 50 years
warrants further investigation of the specic circumstances
and cause of the death. A family history of cardiomyopathy,
placement of an implantable cardioverter-debrillator or
pacemaker, long QT syndrome (or congenital deafness,
which has been associated with long QT syndrome), or
congenital heart disease should prompt evaluation for a
potential heritable cardiac cause for the syncope. A family
history of syncope may also be helpful because multiple
members in the same family may have reex syncope.

Pediatrics in Review
Downloaded from http://pedsinreview.aappublications.org/ by guest on April 2, 2016

Signs and Symptoms


Distinguishing presyncope from true syncope is important.
Presyncope involves symptoms such as weakness, sweating
or a warm rushing feeling, dizziness, visual disturbances,
headache, palpitations, abdominal discomfort/nausea, or
pallor without a complete loss of consciousness. Although
these symptoms frequently precede true syncope, they are
not as concerning when true complete loss of consciousness
does not occur. If a patient did not become completely
accid or remembers all the events that happened (including falling to the ground), he or she may not have experienced true syncope and may require further evaluation to
determine the underlying cause of presyncopal signs and
symptoms.

Laboratory and Imaging Studies


Electrocardiography (ECG) is important in the evaluation of
patients who have syncope because ndings may suggest an
underlying cardiac cause that may not otherwise be evident.
When evaluating the ECG, the clinician must measure the
QT interval. Because the QT interval varies with heart rate, it
must be adjusted to obtain a corrected QT interval (QTc).
The most commonly used equation is Bazetts formula (the
QT interval in seconds divided by the square root of the
previous QRS-to-QRS interval in seconds). The upper limit
of normal for a corrected QT interval is 440 milliseconds in
males and 450 milliseconds in females. However, most
individuals with a corrected QT between 440 and 470
milliseconds with a normal T wave morphology do not have
an underlying cardiac pathology and are not at risk for lifethreatening arrhythmias. QTc values greater than 470 milliseconds should prompt an evaluation for long QTsyndrome
and values greater than 500 milliseconds are almost never
seen in the absence of a pathologic condition. The onset of
the QRS should be scrutinized for the pre-excitation delta
wave seen in Wolff-Parkinson-White syndrome (Fig 5). Other
important ndings are atrioventricular conduction disturbances or atrioventricular block, evidence of ventricular hypertrophy, or other abnormalities that may suggest underlying
heart disease (eg, T-wave inversion in leads V5 and V6, which
may indicate left ventricular pathology).

Figure 5. Pre-excitation: The rst 3 beats show the characteristic delta


wave seen in Wolff-Parkinson-White with the last 3 beats (after the *)
showing normal conduction.

Routine blood studies rarely are helpful in determining


the cause of syncope unless the physical examination or
history suggests a specic underlying cause. Hypoglycemia
can cause syncope, but serum glucose values have frequently returned to normal at the time of evaluation. Assessing glucose concentrations in close proximity to the
syncopal episode may be helpful but often has minimal
value. Serum electrolytes, including calcium, are also of little
utility in establishing a cause of syncope. Although iron
deciency anemia has been implicated in autonomic dysfunction and reex syncope, it is a rare primary cause of
syncope, and a complete blood cell count is typically not
helpful in establishing a diagnosis. In adolescent females,
screening for pregnancy may be indicated because the
physiologic adaptations of pregnancy may result in a syncopal event. A serum or urine drug screen should also be
considered. In the absence of a neurologic cause or focal
neurologic ndings, imaging of the brain with computed
tomography scan or magnetic resonance imaging is rarely
helpful, although these frequently are performed. Electroencephalography is also rarely indicated and may yield
abnormal results immediately after an episode of reex
syncope, which can create the impression of an underlying
seizure disorder.

Diagnostic Tests
One means of potentially assessing for reex syncope is a
head-upright tilt table study, but its routine use is controversial, and results in most instances of routine syncope are
of limited value. In this study, the patient lies on a table that
can be rotated from 0 to 90 degrees from the horizontal
position. During this time, the patient is monitored with a
continuous ECG tracing and blood pressure readings.
Recordings are taken with the patient supine and when
the patient is angled to 60 to 90 degrees by tilting the table.
A positive test result consists of reproduction of symptoms
usually accompanied by hypotension or bradycardia. Other
methods to assess for autonomic dysfunction (such as the
response to a Valsalva maneuver or response to facial
immersion in ice water) can be performed during the
evaluation. The sensitivity and specicity of tilt table testing
varies widely, depending on the protocol being used. The
false-negative rate ranges from 14% to 30% but can be
higher, depending on the protocol. (5) Using different
protocols for the study can increase the sensitivity but
subsequently decrease the specicity, resulting in up to a
45% false-positive rate in some cases. (6)(7) Because patient
history can diagnose most cases of reex syncope, a tilt table
study may not be necessary in all patients, and most pediatric
electrophysiologists do not believe that it is benecial in the

Vol. 37 No. 4
Downloaded from http://pedsinreview.aappublications.org/ by guest on April 2, 2016

APRIL 2016

165

diagnostic evaluation of syncope. (8) However, in cases of


psychogenic syncope, it can be helpful to observe episodes of
syncope that have no heart rate or blood pressure changes.
Tilt table studies may also be useful in determining a cause of
recurrent syncope that does not have clear antecedents or in
cases with atypical symptomatology. Although a formal tilt
table test may not be necessary, monitoring a patients heart
rate and blood pressure in the supine position and after
standing for 3 minutes may be benecial.
If the clinician is concerned about an arrhythmic mechanism of syncope, documenting the heart rhythm during an
episode of syncope is important and can be accomplished by
several different methods. An event monitor is a small
electronic device that can record an ECG when activated.
There are two types of ambulatory event monitors. For the
rst type, the patient wears leads that are continuously
connected to a device that records the heart rhythm. When
the device is activated, it documents the rhythm recorded
several minutes before and after activation. For the second
monitor type, the patients rhythm is recorded by a monitor
that is connected to the patient during episodes of syncope.
Another method for recording a patients heart rhythm is
the 24-hour Holter monitor, which is useful if the syncopal
episodes occur on a daily basis.
In extreme cases, hospitalization with continuous heart
rhythm, blood pressure, and/or ECG monitoring may be
indicated. Another diagnostic approach is using an ECG
monitor that is implanted under the skin (implantable loop
recorder), which can monitor the rhythm continuously for
periods of up to 3 years. With this approach, the patient
wirelessly triggers an ECG recording through an activator
button when he or she feels symptomatic. In addition, the
device automatically records heart rates above or below a
predetermined set rate or if there is a rhythmic pause of
more than 3 seconds. This approach has been shown to be
very effective in determining arrhythmic or reex causes of
syncope. However, it is rarely indicated, except when a
cardiac arrhythmia is suspected but cannot be documented
by other means. (9) Continuous ambulatory blood pressure
monitoring also can be used to document hypotension.
An exercise stress test (bicycle or treadmill) is indicated
for episodes of syncope that occur during exercise. The heart
rate and blood pressure response to exercise can help
determine if there is a reex cause of syncope. In addition,
arrhythmias during exercise may be observed. In a patient with catecholamine-sensitive polymorphic ventricular
tachycardia, the baseline physical examination and ECG
ndings are normal, but the patient develops increasing
ventricular ectopy and frequently ventricular tachycardia
during exercise. Although ischemic changes due to coronary

166

artery abnormalities can develop during exercise stress testing, this modality is not sensitive for detecting ischemia in
pediatrics, and denitive imaging of the coronary arteries
and ventricular function with echocardiography, computed
tomography scan, or magnetic resonance imaging or cardiac
catheterization is indicated if there is concern for a coronary
artery abnormality.
Single isolated episodes of syncope with a history typical
for reex mechanism syncope do not require extensive
evaluation. Multiple episodes of syncope or syncope with
unusual historical details not classic for reex syncope may
require more extensive evaluation, including blood laboratory tests, neurologic imaging, or subspecialty consultation.

MANAGEMENT
An important part of management is deciding who should
be referred to pediatric cardiology or neurology. If there are
specic concerns for a neurologic or particularly a potentially life-threatening cardiologic cause, the referral should
be immediate. Any patient with a syncopal episode who
required resuscitation by CPR or received debrillation
from an automated external debrillator or debrillator in
the hospital should undergo a thorough evaluation by a
pediatric cardiologist before leaving the hospital or clinic.
Fundamental to the treatment of reex syncope is
increasing water and salt intake, particularly upon waking.
A regular exercise program is also very important in therapy
of reex syncope. Exercise is frequently challenging because
patients may fatigue easily and feel exhausted after minimal
exertion. If exercise is not continued, despite its difculty,
patients can become more deconditioned, which exacerbates the underlying mechanism of their symptoms. Patients
should also avoid diuretics such as caffeine and alcohol. They
should be instructed to lie down with their feet elevated when
symptoms start or perform counterpulsation measures in an
attempt to avoid syncope. Counterpulsation measures involve
purposeful contraction of the muscles in the arms and legs in
an effort to mechanically force blood return to the heart.
Patients should also avoid circumstances that predispose to
syncope, such as dehydration or prolonged standing.
Medications may be considered for patients with refractory reex syncope. However, medications are frequently
ineffective and may have adverse effects. In addition, they
may improve symptoms but only rarely completely eliminate them. One of the more effective medications is
midodrine. This a-receptor agonist constricts arterioles
and veins and increases peripheral vascular resistance.
Oral midodrine is rapidly absorbed. The dose is normally
2.5 to 10 mg. The drug reaches peak blood concentrations

Pediatrics in Review
Downloaded from http://pedsinreview.aappublications.org/ by guest on April 2, 2016

in 1 to 2 hours and the half-life is approximately 3 to 4 hours.


For this reason, midodrine is usually taken 3 times a day.
However, the dosing schedule should be shifted toward
earlier in the day because the blood pressure-elevating
effects are not desirable when the patient is supine at
bedtime. The typical dosing schedule is to take 3 doses
4 hours apart beginning in the morning (eg, 7 AM, 11 AM,
and 3 PM). Because midodrine may increase blood pressure
substantially, several blood pressure readings should be
obtained 1 to 2 hours following the initial dose. Other
adverse effects include blurred vision; headache; pounding
in the ears; burning; itching; chills; piloerection; and
urinary frequency, retention, or urgency.
b-blockers have also been used but are ineffective in
many patients. (10) In fact, some patients with reex syncope actually feel worse when receiving these medications.
b-blockers may also cause fatigue and should be used with
caution in patients who have diabetes (because they may
block hypoglycemic symptoms) or reactive airways disease.
b-blocker therapy is frequently initiated with a very low dose;
even subtherapeutic doses may be sufcient to reduce
symptoms. The dose can be slowly titrated upward to achieve
the desired effect. Fludrocortisone is a synthetic corticosteroid that acts similarly to aldosterone, causing sodium and
water retention and increasing potassium excretion. In theory, udrocortisone should increase the blood volume (and,
thus, decrease venous pooling and symptoms) in patients
with reex syncope, thereby improving symptoms. However,
udrocortisone was ineffective in a double-blind, placebocontrolled study in children with reex syncope. (11)
Although reex syncope may result in heart rate pauses
greater than 10 to 20 seconds, a pacemaker is almost never
indicated. In rare cases where severe bradycardia is the
overwhelmingly predominant symptom or in small children
with multiple breathholding spells occurring on a daily or
weekly basis, a pacemaker may be benecial. However,
caution should be exercised in teenage patients because
the reex syncope response is rarely only bradycardia, and a
pacemaker may not be benecial for patients in whom both
hypotension and bradycardia play a role in their syncope.

they should be warned that their syncope may recur and be


instructed to lie down with their feet up if they feel dizzy.
Syncope is a very frightening event for most families, and
they frequently need signicant reassurance about its
benign nature. Many patients with recurrent reex syncope
improve after 1 to 2 years or in late adolescence and may
even have their symptoms completely resolve, although
patients may have recurrences into their third decade.
Recurrent syncope frequently responds to conservative
measures such as diet and exercise. However, patients
may require medication for even a few years until their
symptoms improve. For some patients, the reex syncope
can become a debilitating condition with a signicant
component of depression. For these patients, a chronic pain
rehabilitation program that includes psychiatric behavioral
therapy may be benecial. Clinicians must explain to the
family that there is no quick cure and that complete
resolution of reex syncope and presyncope symptoms is
frequently not possible. The goal is to minimize symptoms
so that patients can be functional in their everyday lives.

Summary
On the basis of strong evidence, syncope is a typically benign
entity in young patients and usually has a good prognosis. (3)
On the basis of good evidence and expert opinion, syncope
during active exercise, syncope that occurs without warning, and
syncope preceded by severe chest pain or rapid palpitations
possibly have cardiac origins and require further evaluation. (12)
On the basis of good evidence and expert opinion, most causes of
syncope can be determined by a thorough history, but
electrocardiography and physical examination are also important
parts of the evaluation to rule out cardiac causes. (1)
On the basis of good evidence and expert opinion, the primary
therapies for reex syncope are conservative measures such as
increased uid and salt intake, a routine exercise program, and
counterpulsation measures. (13)
On the basis of good evidence, medications are not particularly
effective at preventing recurrent episodes of syncope, with the
exception of midodrine, which may be benecial in children with
a prominent hypotensive response. (14)

PROGNOSIS
Patients who have a single episode of reex syncope with
normal physical examination ndings, family history, and
ECG results require no further evaluation or follow-up, but

CME quiz, references, and suggested readings for this article are at
http://pedsinreview.aappublications.org/content/37/4/159.

Vol. 37 No. 4
Downloaded from http://pedsinreview.aappublications.org/ by guest on April 2, 2016

APRIL 2016

167

PIR Quiz
There are two ways to access the journal CME quizzes:
1. Individual CME quizzes are available via a handy blue CME link under the article title in the Table of Contents of any issue.
2. To access all CME articles, click Journal CME from Gateways orange main menu or go directly to: http://www.
aappublications.org/content/journal-cme.

1. The autonomic reex responsible for vasovagal syncope results in which of the following
physiological changes?
A.
B.
C.
D.
E.

Bradycardia, vasoconstriction, and hypertension.


Bradycardia, vasodilation, and hypertension.
Bradycardia, vasodilation, and hypotension.
Tachycardia, vasoconstriction, and hypertension.
Tachycardia, vasodilation, and hypotension.

2. A 10-year-old boy presents to the emergency department following a period of loss of


consciousness. His parents report that he was playing tag with his brother when he
suddenly dropped to the ground. The episode lasted less than 1 minute, after which time
he quickly returned to baseline activity. He does not describe any aura or symptoms before
the event and there were no abnormal movements during the event. Which of the
following diagnoses is most likely?
A.
B.
C.
D.
E.

Cardiogenic syncope.
Migraine.
Psychogenic syncope.
Seizure.
Vasovagal syncope.

3. A 5-year-old girl was standing in line at a grocery store with her mother when she suddenly
fell to the ground. Her mother reports that the girl had shaking of her arms and legs that
lasted for 20 seconds and urinary incontinence. Following this event, she was disoriented
and sleepy for 1 hour. Which of the following characteristics is most suggestive of seizure
as a cause for her loss of consciousness?
A. The episode is followed by a 1-hour postictal period characterized by disorientation
and sleepiness.
B. The episode is associated with urinary incontinence.
C. The episode is described as shaking of her arms and legs.
D. The episode occurred in the absence of physical activity.
E. The episode is 20 seconds long.
4. A 14-year-old girl presents to your clinic with 5 episodes of syncope over the last 2 months.
These episodes usually occur before school but without specic triggers. Her episodes of
loss of consciousness typically last 20 to 30 minutes, after which time she quickly returns to
baseline activity. On review of systems, she has a 2-year history of abdominal pain,
headache, and intermittent blurred vision, which have resulted in many missed days of
school over the last 3 months. What should your next step be?
A.
B.
C.
D.
E.

REQUIREMENTS: Learners
can take Pediatrics in
Review quizzes and claim
credit online only at:
http://pedsinreview.org.
To successfully complete
2016 Pediatrics in Review
articles for AMA PRA
Category 1 CreditTM,
learners must
demonstrate a minimum
performance level of 60%
or higher on this
assessment, which
measures achievement of
the educational purpose
and/or objectives of this
activity. If you score less
than 60% on the
assessment, you will be
given additional
opportunities to answer
questions until an overall
60% or greater score is
achieved.
This journal-based CME
activity is available
through Dec. 31, 2018,
however, credit will be
recorded in the year in
which the learner
completes the quiz.

Order electroencephalography.
Order magnetic resonance imaging.
Order a tilt table study.
Refer her to psychiatry.
Tell her this is due to a virus and that it will get better.

5. Which of the following is an inappropriate recommendation for patients with reex/


neurally mediated syncope?
A.
B.
C.
D.
E.

168

Avoid all physical activity.


Avoid caffeine and alcohol.
Avoid periods of prolonged standing.
Increase salt and water intake.
Lie down with feet elevated to treat feelings of lightheadedness.

Pediatrics in Review
Downloaded from http://pedsinreview.aappublications.org/ by guest on April 2, 2016

Syncope
Bryan Cannon and Philip Wackel
Pediatrics in Review 2016;37;159
DOI: 10.1542/pir.2014-0109

Updated Information &


Services

including high resolution figures, can be found at:


http://pedsinreview.aappublications.org/content/37/4/159

References

This article cites 16 articles, 3 of which you can access for free at:
http://pedsinreview.aappublications.org/content/37/4/159#BIBL

Subspecialty Collections

This article, along with others on similar topics, appears in the


following collection(s):
Medical Education
http://beta.pedsinreview.aappublications.org/cgi/collection/medical_
education_sub
Journal CME
http://beta.pedsinreview.aappublications.org/cgi/collection/journal_c
me
Adolescent Health/Medicine
http://beta.pedsinreview.aappublications.org/cgi/collection/adolescen
t_health:medicine_sub
Sports Medicine/Physical Fitness
http://beta.pedsinreview.aappublications.org/cgi/collection/sports_m
edicine:physical_fitness_sub
Cardiology
http://beta.pedsinreview.aappublications.org/cgi/collection/cardiolog
y_sub
Cardiovascular Disorders
http://beta.pedsinreview.aappublications.org/cgi/collection/cardiovas
cular_disorders_sub

Permissions & Licensing

Information about reproducing this article in parts (figures, tables) or


in its entirety can be found online at:
http://beta.pedsinreview.aappublications.org/site/misc/Permissions.x
html

Reprints

Information about ordering reprints can be found online:


http://beta.pedsinreview.aappublications.org/site/misc/reprints.xhtml

Downloaded from http://pedsinreview.aappublications.org/ by guest on April 2, 2016

Syncope
Bryan Cannon and Philip Wackel
Pediatrics in Review 2016;37;159
DOI: 10.1542/pir.2014-0109

The online version of this article, along with updated information and services, is
located on the World Wide Web at:
http://pedsinreview.aappublications.org/content/37/4/159

Pediatrics in Review is the official journal of the American Academy of Pediatrics. A monthly
publication, it has been published continuously since 1979. Pediatrics in Review is owned,
published, and trademarked by the American Academy of Pediatrics, 141 Northwest Point
Boulevard, Elk Grove Village, Illinois, 60007. Copyright 2016 by the American Academy of
Pediatrics. All rights reserved. Print ISSN: 0191-9601.

Downloaded from http://pedsinreview.aappublications.org/ by guest on April 2, 2016

You might also like